Косинус пи 2: Таблица значений тригонометрических функций

Содержание

Формулы приведения тригонометрических функций

Формулы приведения тригонометрических функций

      Формулы привидения тригонометрических функций представлены в виде таблицы. Ниже находятся некоторые формулы приведения в табличном виде. Еще ниже эти формулы приведения расписаны для некоторых тригонометрических функций в виде тригонометрических тождеств.Таблицы значений тригонометрических функций находятся на другой странице.

      Формулы привидения для синуса выглядят так:

sin (π/2 + α) = cos α

sin (π + α) = — sin α

sin (3π/2 + α) = — cos α

sin (π/2 — α) = cos α

sin (π — α) = sin α

sin (3π/2 — α) = — cos α

sin (2π — α) = — sin α

      Формулы приведения для тригонометрической функции синус будут следующие. Синус угла пи пополам (пи/2) плюс или минус угол альфа равняется косинусу угла альфа. Синус угла пи плюс угол альфа или угла два пи минус альфа по формуле приведения будет равен минус синусу угла альфа. Синус угла три вторых пи (3пи/2) плюс или минус альфа равен минус косинусу альфа. Если угол равняется пи минус альфа, то синус такого угла равняется синусу угла альфа. Тригонометрия формулы и тригонометрические тождества. Перевод синуса в косинус.

      Тригонометрические формулы привидения для косинуса приобретают такой вид:

cos (π/2 + α) = — sin α

cos (π + α) = — cos α

cos (3π/2 + α) = sin α

cos (π/2 — α) = sin α

cos (π — α) = — cos α

cos (3π/2 — α) = — sin α

cos (2π — α) = cos α

      Для косинуса формулы приведения имеют следующий вид. Косинус угла пи пополам (пи/2) плюс угол альфа и косинус три вторых пи минус альфа равняются минус синусу угла альфа. Косинус угла пи плюс или минус угол альфа в результате равен минус косинусу альфа. Формула приведения для косинуса угла (3пи/2) три вторых пи плюс угол альфа и пи пополам минус альфа равняется синусу угла альфа. Тригонометрическая функция косинус пи пополам (1/2 пи) минус альфа равняется тригонометрической функции косинус угла альфа. Тригонометрия формулы. Перевод косинуса в синусы.

      Тригонометрическая функция тангенс имеет следующие формулы привидения:

tg (π/2 + α) = — ctg α

tg (π + α) = tg α

tg (3π/2 + α) = — ctg α

tg (π/2 — α) = ctg α

tg (π — α) = — tg α

tg (3π/2 — α) = ctg α

tg (2π — α) = tg α

      Тригонометрические формулы приведения функции тангенс tg. Тангенс угла пи деленное на два плюс альфа и угла три вторых пи плюс альфа приравниваются минус котангенсу угла альфа. Тригонометрическая функция тангенс угла пи на два или три пи деленное на два (3/2 пи) минус альфа равна котангенсу альфа. Тангенс угла пи минус альфа равен минус тангенсу угла альфа. Формулы приведения для тангенса пи плюс альфа и тангенса два пи минус альфа равняются тангенсу угла альфа. Тригонометрические тождества. Перевод тангенса в котангенсы.

      Тригонометрическая функция тангенс имеет следующие формулы привидения:

ctg (π/2 + α) = — tg α

ctg (π + α) = ctg α

ctg (3π/2 + α) = — tg α

ctg (π/2 — α) = tg α

ctg (π — α) = — ctg α

ctg (3π/2 — α) = tg α

ctg (2π — α) = ctg α

      Формулы приведения функции котангенс ctg угла. Для угла 1/2 пи плюс альфа и угла 3/2 пи плюс альфа котангенс равняется минус тангенсу -tg угла альфа. Если в этих же выражениях угол альфа не прибавляется, а вычитается, тогда котангенс такого угла равняется тангенсу угла альфа. Функция котангенс пи минус альфа равна минус котангенсу угла альфа. Котангенс угла пи плюс альфа и 2 пи минус альфа будет равен котангенсу угла альфа. Тригонометрические тождества и формулы тригонометрия. Перевод котангенсов в тангенсы.

Если вам понравилась публикация и вы хотите знать больше, помогите мне в работе над другими материалами.

      17 января 2010 года — 1 декабря 2018 года..

© 2006 — 2021 Николай Хижняк. Все права защищены.

Таблица значений тригонометрических функций

Примечание. В данной таблице значений тригонометрических функций используется знак √ для обозначения квадратного корня. Для обозначения дроби — символ «/».

См. также полезные материалы:

Для определения значения тригонометрической функции, найдите его на пересечении строки с указанием тригонометрической функции. Например, синус 30 градусов — ищем колонку с заголовком sin (синус) и находим пересечение этой колонки таблицы со строкой «30 градусов», на их пересечении считываем результат — одна вторая. Аналогично находим косинус 60 градусов, синус 60 градусов (еще раз, в пересечении колонки  sin (синус) и строки 60 градусов находим значение sin 60 = √3/2 ) и т.д. Точно так же находятся значения синусов, косинусов и тангенсов других «популярных» углов.

Синус пи, косинус пи, тангенс пи и других углов в радианах

Приведенная ниже таблица косинусов, синусов и тангенсов также подходит для нахождения значения тригонометрических функций, аргумент которых задан в радианах. Для этого воспользуйтесь второй колонкой значений угла. Благодаря этому можно перевести значение популярных углов из градусов в радианы. Например, найдем угол 60 градусов в первой строке и под ним прочитаем его значение в радианах. 60 градусов равно π/3 радиан.

Число пи однозначно выражает зависимость длины окружности от градусной меры угла. Таким образом, пи радиан равны 180 градусам. 

Любое число, выраженное через пи (радиан) можно легко перевести в градусную меру, заменив число пи (π) на 180.

Примеры:
1. Синус пи
sin π = sin 180 = 0
таким образом, синус пи — это тоже самое, что синус 180 градусов и он равен нулю.

2. Косинус пи.
cos π = cos 180 = -1
таким образом, косинус пи — это тоже самое, что косинус 180 градусов и он равен минус единице.

3. Тангенс пи
tg π = tg 180 = 0

таким образом, тангенс пи — это тоже самое, что тангенс 180 градусов и он равен нулю.

Таблица значений синуса, косинуса, тангенса для углов 0 — 360 градусов (часто встречающиеся значения)  



Если в таблице значений тригонометрических функций вместо значения функции указан прочерк (тангенс (tg) 90 градусов, котангенс (ctg) 180 градусов) значит при данном значении градусной меры угла функция не имеет определенного значения. Если же прочерка нет — клетка пуста, значит мы еще не внесли нужное значение. Мы интересуемся, по каким запросам к нам приходят пользователи и дополняем таблицу новыми значениями, несмотря на то, что текущих данных о значениях косинусов, синусов и тангенсов самых часто встречающихся значений углов вполне достаточно для решения большинства задач. 

Таблица значений тригонометрических функций sin, cos, tg для наиболее популярных углов


 0, 15, 30, 45, 60, 90 … 360 градусов  
(цифровые значения «как по таблицам Брадиса»)  
значение угла α (градусов)  значение угла α в радианах  sin (синус)  cos (косинус)  tg (тангенс)  ctg (котангенс) 
0 0

0

1

0

-

15

π/12

0,2588

0,9659

0,2679

3,7321

30

π/6

0,5000

0,8660

0,5774

1,7321

45

π/4

0,7071

0,7071

1

1

50

5π/18

 0,7660

0,6428

1.1918

0,8391

60

π/3

0,8660

0,5000

1,7321

0,5774

65

13π/36

0,9063

0,4226

2,1445

0,4663

70

7π/18

0,9397

0,3420

2,7475

0,3640

75

5π/12

0,9659

0,2588

3,7321

0,2679

90

π/2

1

0

-

0

105

 5π/12

0,9659

-0,2588

-3,7321

-0,2679

120

2π/3

0,8660

-0,5000

-1,7321

-0,5774

135

3π/4

0,7071

-0,7071

-1

-1

140

7π/9

 0,6428

-0,7660

-0,8391

-1,1918

150

5π/6

0,5000

-0,8660

-0,5774

-1,7321

180

π

0

-1

0

-

270

3π/2

-1

0

-

0

360

0

1

0

-

 Иногда для быстрых расчетов нужно не точное, а вычисляемое значение (число десятичной дробью), которое раньше искали в таблицах Брадиса. Поэтому, в дополнение к таблице точных значений тригонометрических функций приведены эти же самые значения, но в виде десятичной дроби, округленной до четвертого знака. Дополнительно в таблицу включены «нестандартные» значения тангенса, косинуса, синуса 140 градусов, синуса 105, 70, косинуса 105 и 50 градусов.


 Начать курс обучения

2 Пи или не 2 Пи — вот в чём вопрос / Блог компании Wolfram Research / Хабр


Перевод поста Giorgia Fortuna «2 Pi or Not 2 Pi?».
Выражаю огромную благодарность Кириллу Гузенко за помощь в переводе.

Три месяца назад мир (или по крайней мере мир гиков) праздновал день Пи (03.14.15…). Сегодня (6/28 — 28 июня 2015 г.) другой математический день — день 2π, или

день Тау

(2π = 6.28319…).

Некоторые говорят, что день тау действительно является днём для празднования, и что τ (= 2π), а не π, должен быть самой важной константой. Все началось в 2001 году со вступительного слова знаменитого эссе Боба Пале, математика из университета Юты:

“Я знаю, что некоторые сочтут это богохульством, но я считаю, что π — это ошибка”.

Это вызвало в некоторых кругах празднование дня тау — или, как многие говорят, единственного дня, в который можно съесть два пи(рога) (2pies≈2π — игра слов в англ. языке).

Однако правда ли то, что τ — константа получше? В современном мире это довольно просто проверить, а Wolfram Language делает эту задачу ещё проще (действительно, недавний пост в блоге Майкла Тротта о датах в числе пи, вдохновлённый постом Стивена Вольфрама о праздновании векового дня числа пи, весьма активно задействовал Wolfram Language). Я начала с рассмотрения 320000 препринтов на arXiv.org чтобы посмотреть, сколько в действительности формул содержат 2π по сравнению с теми, что содержат просто π или π с другими сомножителями.

Вот облако из некоторых формул, построенное с помощью функции WordCloud, содержащих 2π:



Я обнаружила, что лишь 18% рассматриваемых формул содержат 2π, из чего следует, что перейти на использование τ — не лучший выбор.

Но почему тогда сторонники использования τ считают, что мы должны перейти к использованию этого нового символа? Одна из причин заключается в том, что использование τ должно сделать тригонометрию проще для изучения и понимания. В конце концов, в тригонометрии мы используем не углы, а радианы, а в окружности содержится 2π радиан. Это означает, что четверти круга соответствует 1/2π радиан, или π/2, а не четверть чего-то! От этой несправедливости можно избавиться введением символа τ, и тогда каждой части окружности будет соответствовать такая же часть от τ. Например, четверти окружности соответствовал бы угол τ/4.

Лично у меня использование числа π не вызывает каких-то сильных негативных чувств, и честно говоря, я не думаю, что использование τ позволило бы студентам быстрее изучать тригонометрию. Давайте вспомним о двух самых важных тригонометрических функциях — синусе и косинусе. Пожалуй, самые важные в изучении тригонометрии формулы — sin= cos(2π) = 1, и sin() = cos(π) = –1. Я не только всегда предпочитала использовать косинус потому, что его значения легче запомнить (нет никаких дробных значений в π и 2π), но я и также всегда помнила, что синус и косинус отличаются тем, что одна функция принимает ненулевые значения в точках, кратных π, а другая принимает ненулевые значения в дробных частях π. Если использовать τ, то мы потеряем эту симметрию, и у нас будут уравнения sin = cos(τ) = 1 и sin = cos = –1.

Учитывая вышесказанное, получается, что использование τ или π есть вопрос личного предпочтения. Это справедливое заключение, однако нам нужен более строгий подход для определения того, какая из констант более полезна.

Даже тот подход, которым я руководствовалась вначале, может привести к неправильным выводам. В Тау манифесте Майкл Хартл приводит некоторые примеры тех мест, где часто можно встретить 2π:

И в самом деле, все эти формулы выглядели бы проще, если бы мы использовали τ. Однако это всего лишь шесть формул из того огромного количества, которые ученые регулярно используют, и как я упоминала ранее, не так уж много математических выражений содержат 2π. Тем не менее, вполне возможно, что формулы, не содержащие 2π, будут более простыми, если будут записаны через τ. Например, выражение 4π² запишется просто как τ².

Поэтому я вернулась к научным статьям, чтобы выяснить, сделает ли использование τ вместо 2π (и τ/2 вместо π) формулы более простыми. Например, вот те, которые станут более простыми с использованием τ:

А вот некоторые из тех, которые не станут:

Позвольте объяснить, что я подразумеваю под более простой формой записи на примере: если я возьму часть, содержащую π в нижней левой формуле таблицы с формулами Тау манифеста (см. выше):

Я могу заменить π на τ/2 с помощью функции ReplaceAll и получить:

Посмотрев на эти два выражения, можно увидеть, что второе проще. И дело здесь не в интуиции — во втором просто меньше символов. Для большей ясности можно рассмотреть соответствующие им древовидные графы посредством функции TreeForm:

Для получения численного представления их различия мы можем использовать количества ветвей дерева, которые соответствуют числу символов в исходных формулах:

Чтобы определить, упрощается ли формула в результате использования τ, я вычислила сложность каждой формулы (которая определяется количеством ветвей дерева), содержащей π, для формул из статей, в зависимости от того, какая из констант используется — π или τ. Для большей точности я сначала удалила все выражения, которые были равны или эквивалентны π или 2π. Я чувствовала, что будет несправедливо их учитывать, потому что они часто встречаются сами по себе, вне формул. Затем я сравнила случаи, когда использование τ упрощало формулу с теми, когда усложняло, и лишь 43% формул стали проще с использованием τ, то есть в более чем половине случаев использование τ усложняет формулу. Иными словами, из этого сравнения следует, что мы должны продолжать использовать π. Тем не менее, это не конец истории.

Я заметила вот что: если выражение становится более или менее сложным, то это значит, что количество ветвей у него менее 40. В самом деле, если посмотреть на процент формул, которые становятся проще при использовании π или τ и имеют количество ветвей меньше определённого значения, то вы увидите следующую картину:

Ось х представляет верхнюю границу количества ветвей. Из этого следует, что почти для всех формул их сложность зависит от выбора символа только в случае, если число ветвей меньше 50.

Более важное наблюдение заключается в том, что по мере роста сложности формулы ситуация резко меняется. Даже если выбрать формулы со сложностью большей, чем 3, как рассмотренная ранее формула , то тогда лишь 48% формул станут проще с использованием π против 52% для τ. Приведенные ниже графики показывают, как процентные отношения формул, которые проще с использованием π или τ, изменяются в зависимости от сложности:

Как можно заметить, при числе ветвей более 48 графики начинают вести себя хаотично. Это следствие того, что лишь 0,4% формул выборки имеют сложность более 50. Мы ничего особо конкретного не можем сказать о них, и прошлый опыт говорит нам о том, что это нам очень-то и не нужно.

И из этого графика также следует то, что в повседневной жизни и для каких-то выражений, которые сложнее чего-то наподобие , в целях упрощения выражений нам однозначно следует использовать τ. Но есть еще один момент, которого я не коснулась. Что насчёт различных областей приложений?

Возможно, в физике формулы будут проще выглядеть с τ, а в других областях — нет. Изначально я включила в поиск статьи из различных областей; однако, я не проверяла принадлежность формул, содержащих π, тем или иным областям знаний, а также то, принадлежат ли формулы, которые становятся проще с использованием τ, какому-то ограниченному подмножеству областей. В самом деле, если рассмотреть лишь математические статьи, то результат окажется следующим:

Получается, что лишь 23% всех формул становятся проще с использованием τ, да и то лишь для довольно сложных выражений. Вот что-то наподобие этого:

можно проще записать через τ, однако большинство подобных выражений встречается весьма редко. Получается, что либо учёные из различных областей должны использовать различные соглашения в зависимости от специфичных для своих областей формул, либо все должны перейти на использование τ, хотя на самом деле для некоторых областей это не имеет особого смысла. В конце концов, демократия предполагает удовлетворённость большинства, и невозможно угодить всем без исключения.

Тем не менее, вышеуказанная формула содержит ещё кое-что, на чём я бы хотела заострить внимание. Так она выглядит с τ:

Пускай выражение действительно проще записывается через τ, однако подобное улучшение столь незначительно, что становится пренебрежимо малым. Рассмотрим, например, эти два выражения вместе с количествами их ветвей:

И соответствующие им выражения в τ:

Первая формула проще в τ, но количество ветвей становится лишь на 1/13 меньше по сравнению с первоначальным количеством, в то время как второе выражение проще записывается в π, а после замены его сложность возрастает на 1/6. Другими словами, улучшение в первом случае составило 1/13, а во втором -1/6 (знак минус означает ухудшение). Среднее значение вектора составляет -0.044 — отрицательное число, что означает, что использование τ в этих двух выражениях делает общий вектор на 0,044 хуже.

Подобный векторный подход отличается от ранее использованного подхода, при котором не учитывался размер уравнения. В нём считается количество улучшений, а не количество упрощенных выражений, и это переворачивает с ног на голову предыдущие выводы. Я получила эти векторы для формул, в которых сложность ограничена снизу — всё так же, как и в предыдущем примере. Получается, что общее улучшение при замене π на τ уменьшается с увеличением сложности:

а наименьшее ухудшение -0,04 достигается при сложности 5. Как можно заметить, общее улучшение всегда отрицательно; это означает, что пусть и большее количество формул имеют более короткую запись через τ (в зависимости от области), но в целом сумма всех «упрощений» формул перевешивается всеми «усложнениями».

В итоге всего этого исследования у меня сформировалась такая позиция: думаю, нам стоит быть довольными нашим старым другом π и не переходить на использование τ.

У меня есть два заключительных замечания. Первое заключается в том, что если бы мы жили в мире, где активнее используется τ, то вывод был бы полностью противоположным. Если бы наши выражения уже записывались бы через τ, и мы исследовали бы вопрос о переходе на использование π и вопросы упрощения, то наш график сумм векторов выглядел бы следующим образом:

Подобное различие объясняется тем, что векторы, которые используются для построения графиков, зависят от исходных сложностей, и потому меняются при изменении оных.

Из этого следует, что для большинства формул, которые имеют сложность больше двух и меньше 18, улучшение от замены τ на π будет отрицательным. К сожалению для сторонников τ, мы живем всё таки в мире π.

Второе замечание, на которое навёл меня Майкл Тротт, заключается в том, что 2/3 из формул, указанных в Тау манифесте (зеленая таблица в начале поста), содержат не просто 2π, а комплексное выражение 2πi. Это говорит о том, что, возможно, сама постановка вопроса, на который я пыталась ответить, является некорректной. Быть может, лучшей будет следующая формулировка: будет ли смысл ввести новый символ τ для комплексного числа 2πi?

Это новое обозначение потребует также замены πi на τ/2, но это не повлияет на сложность πi. В общем, формулы, содержащие πi, либо уменьшат, либо сохранят свою сложность. Вот облако формул, которые станут проще:

Так они станут выглядеть после подстановки 2πi на τ:

Можно было бы возразить, что процент улучшения формул не будет достаточно высоким, и переход от 2πi к τ неоправданным. Однако факты говорят обратное: из всех формул, содержащих πi, 75% станут проще, а остальные 25% сохранят свой уровень сложности — то есть ни одна формула не станет сложнее. Это весомый аргумент, но я не в том положении, чтобы претворить эту идею; однако, полагаю, что равенство τ = 2πi перспективнее (и менее исторически сложно), чем τ = 2π.

Независимо от вашего мнения касательно τ, надеюсь, что вы прекрасно провели день Тау. Наслаждайтесь сегодняшним днём двух пи(рогов) — мнимых или каких бы то ни было.

Функция ПИ — Служба поддержки Office

В этой статье описаны синтаксис формулы и использование функции ПИ в Microsoft Excel.

Описание

Возвращает число 3,14159265358979 — математическую константу «пи» с точностью до 15 цифр.

Синтаксис

ПИ()

У функции ПИ нет аргументов.

Пример

Скопируйте образец данных из следующей таблицы и вставьте их в ячейку A1 нового листа Excel. Чтобы отобразить результаты формул, выделите их и нажмите клавишу F2, а затем — клавишу ВВОД. При необходимости измените ширину столбцов, чтобы видеть все данные.

Данные

Радиус

3

Формула

Описание

Результат

=ПИ()

Возвращает число «пи».2)

Площадь круга с радиусом, указанным в ячейке A3.

28,27433388

Таблица косинусов (полная, градусы и значения)

В данной таблице представлены значения косинусов от 0° до 360°. Таблица косинусов нужна, чтобы узнать, чему равен косинус угла. Нужно только найти его в таблице. Для начала короткая версия таблицы.

https://uchim.org/matematika/tablica-kosinusov — uchim.org

Таблица косинусов для 0°-180°

cos(1°)0.9998
cos(2°)0.9994
cos(3°)0.9986
cos(4°)0.9976
cos(5°)0.9962
cos(6°)0.9945
cos(7°)0.9925
cos(8°)0.9903
cos(9°)0.9877
cos(10°)0.9848
cos(11°)0.9816
cos(12°)0.9781
cos(13°)0.9744
cos(14°)0.9703
cos(15°)0.9659
cos(16°)0.9613
cos(17°)0.9563
cos(18°)0.9511
cos(19°)0.9455
cos(20°)0.9397
cos(21°)0.9336
cos(22°)0.9272
cos(23°)0.9205
cos(24°)0.9135
cos(25°)0.9063
cos(26°)0.8988
cos(27°)0.891
cos(28°)0.8829
cos(29°)0.8746
cos(30°)0.866
cos(31°)0.8572
cos(32°)0.848
cos(33°)0.8387
cos(34°)0.829
cos(35°)0.8192
cos(36°)0.809
cos(37°)0.7986
cos(38°)0.788
cos(39°)0.7771
cos(40°)0.766
cos(41°)0.7547
cos(42°)0.7431
cos(43°)0.7314
cos(44°)0.7193
cos(45°)0.7071
cos(46°)0.6947
cos(47°)0.682
cos(48°)0.6691
cos(49°)0.6561
cos(50°)0.6428
cos(51°)0.6293
cos(52°)0.6157
cos(53°)0.6018
cos(54°)0.5878
cos(55°)0.5736
cos(56°)0.5592
cos(57°)0.5446
cos(58°)0.5299
cos(59°)0.515
cos(60°)0.5
cos(61°)0.4848
cos(62°)0.4695
cos(63°)0.454
cos(64°)0.4384
cos(65°)0.4226
cos(66°)0.4067
cos(67°)0.3907
cos(68°)0.3746
cos(69°)0.3584
cos(70°)0.342
cos(71°)0.3256
cos(72°)0.309
cos(73°)0.2924
cos(74°)0.2756
cos(75°)0.2588
cos(76°)0.2419
cos(77°)0.225
cos(78°)0.2079
cos(79°)0.1908
cos(80°)0.1736
cos(81°)0.1564
cos(82°)0.1392
cos(83°)0.1219
cos(84°)0.1045
cos(85°)0.0872
cos(86°)0.0698
cos(87°)0.0523
cos(88°)0.0349
cos(89°)0.0175
cos(90°)0
cos(91°)-0.0175
cos(92°)-0.0349
cos(93°)-0.0523
cos(94°)-0.0698
cos(95°)-0.0872
cos(96°)-0.1045
cos(97°)-0.1219
cos(98°)-0.1392
cos(99°)-0.1564
cos(100°)-0.1736
cos(101°)-0.1908
cos(102°)-0.2079
cos(103°)-0.225
cos(104°)-0.2419
cos(105°)-0.2588
cos(106°)-0.2756
cos(107°)-0.2924
cos(108°)-0.309
cos(109°)-0.3256
cos(110°)-0.342
cos(111°)-0.3584
cos(112°)-0.3746
cos(113°)-0.3907
cos(114°)-0.4067
cos(115°)-0.4226
cos(116°)-0.4384
cos(117°)-0.454
cos(118°)-0.4695
cos(119°)-0.4848
cos(120°)-0.5
cos(121°)-0.515
cos(122°)-0.5299
cos(123°)-0.5446
cos(124°)-0.5592
cos(125°)-0.5736
cos(126°)-0.5878
cos(127°)-0.6018
cos(128°)-0.6157
cos(129°)-0.6293
cos(130°)-0.6428
cos(131°)-0.6561
cos(132°)-0.6691
cos(133°)-0.682
cos(134°)-0.6947
cos(135°)-0.7071
cos(136°)-0.7193
cos(137°)-0.7314
cos(138°)-0.7431
cos(139°)-0.7547
cos(140°)-0.766
cos(141°)-0.7771
cos(142°)-0.788
cos(143°)-0.7986
cos(144°)-0.809
cos(145°)-0.8192
cos(146°)-0.829
cos(147°)-0.8387
cos(148°)-0.848
cos(149°)-0.8572
cos(150°)-0.866
cos(151°)-0.8746
cos(152°)-0.8829
cos(153°)-0.891
cos(154°)-0.8988
cos(155°)-0.9063
cos(156°)-0.9135
cos(157°)-0.9205
cos(158°)-0.9272
cos(159°)-0.9336
cos(160°)-0.9397
cos(161°)-0.9455
cos(162°)-0.9511
cos(163°)-0.9563
cos(164°)-0.9613
cos(165°)-0.9659
cos(166°)-0.9703
cos(167°)-0.9744
cos(168°)-0.9781
cos(169°)-0.9816
cos(170°)-0.9848
cos(171°)-0.9877
cos(172°)-0.9903
cos(173°)-0.9925
cos(174°)-0.9945
cos(175°)-0.9962
cos(176°)-0.9976
cos(177°)-0.9986
cos(178°)-0.9994
cos(179°)-0.9998
cos(180°)-1

Таблица косинусов для 181°-360°

cos(181°)-0.9998
cos(182°)-0.9994
cos(183°)-0.9986
cos(184°)-0.9976
cos(185°)-0.9962
cos(186°)-0.9945
cos(187°)-0.9925
cos(188°)-0.9903
cos(189°)-0.9877
cos(190°)-0.9848
cos(191°)-0.9816
cos(192°)-0.9781
cos(193°)-0.9744
cos(194°)-0.9703
cos(195°)-0.9659
cos(196°)-0.9613
cos(197°)-0.9563
cos(198°)-0.9511
cos(199°)-0.9455
cos(200°)-0.9397
cos(201°)-0.9336
cos(202°)-0.9272
cos(203°)-0.9205
cos(204°)-0.9135
cos(205°)-0.9063
cos(206°)-0.8988
cos(207°)-0.891
cos(208°)-0.8829
cos(209°)-0.8746
cos(210°)-0.866
cos(211°)-0.8572
cos(212°)-0.848
cos(213°)-0.8387
cos(214°)-0.829
cos(215°)-0.8192
cos(216°)-0.809
cos(217°)-0.7986
cos(218°)-0.788
cos(219°)-0.7771
cos(220°)-0.766
cos(221°)-0.7547
cos(222°)-0.7431
cos(223°)-0.7314
cos(224°)-0.7193
cos(225°)-0.7071
cos(226°)-0.6947
cos(227°)-0.682
cos(228°)-0.6691
cos(229°)-0.6561
cos(230°)-0.6428
cos(231°)-0.6293
cos(232°)-0.6157
cos(233°)-0.6018
cos(234°)-0.5878
cos(235°)-0.5736
cos(236°)-0.5592
cos(237°)-0.5446
cos(238°)-0.5299
cos(239°)-0.515
cos(240°)-0.5
cos(241°)-0.4848
cos(242°)-0.4695
cos(243°)-0.454
cos(244°)-0.4384
cos(245°)-0.4226
cos(246°)-0.4067
cos(247°)-0.3907
cos(248°)-0.3746
cos(249°)-0.3584
cos(250°)-0.342
cos(251°)-0.3256
cos(252°)-0.309
cos(253°)-0.2924
cos(254°)-0.2756
cos(255°)-0.2588
cos(256°)-0.2419
cos(257°)-0.225
cos(258°)-0.2079
cos(259°)-0.1908
cos(260°)-0.1736
cos(261°)-0.1564
cos(262°)-0.1392
cos(263°)-0.1219
cos(264°)-0.1045
cos(265°)-0.0872
cos(266°)-0.0698
cos(267°)-0.0523
cos(268°)-0.0349
cos(269°)-0.0175
cos(270°)-0
cos(271°)0.0175
cos(272°)0.0349
cos(273°)0.0523
cos(274°)0.0698
cos(275°)0.0872
cos(276°)0.1045
cos(277°)0.1219
cos(278°)0.1392
cos(279°)0.1564
cos(280°)0.1736
cos(281°)0.1908
cos(282°)0.2079
cos(283°)0.225
cos(284°)0.2419
cos(285°)0.2588
cos(286°)0.2756
cos(287°)0.2924
cos(288°)0.309
cos(289°)0.3256
cos(290°)0.342
cos(291°)0.3584
cos(292°)0.3746
cos(293°)0.3907
cos(294°)0.4067
cos(295°)0.4226
cos(296°)0.4384
cos(297°)0.454
cos(298°)0.4695
cos(299°)0.4848
cos(300°)0.5
cos(301°)0.515
cos(302°)0.5299
cos(303°)0.5446
cos(304°)0.5592
cos(305°)0.5736
cos(306°)0.5878
cos(307°)0.6018
cos(308°)0.6157
cos(309°)0.6293
cos(310°)0.6428
cos(311°)0.6561
cos(312°)0.6691
cos(313°)0.682
cos(314°)0.6947
cos(315°)0.7071
cos(316°)0.7193
cos(317°)0.7314
cos(318°)0.7431
cos(319°)0.7547
cos(320°)0.766
cos(321°)0.7771
cos(322°)0.788
cos(323°)0.7986
cos(324°)0.809
cos(325°)0.8192
cos(326°)0.829
cos(327°)0.8387
cos(328°)0.848
cos(329°)0.8572
cos(330°)0.866
cos(331°)0.8746
cos(332°)0.8829
cos(333°)0.891
cos(334°)0.8988
cos(335°)0.9063
cos(336°)0.9135
cos(337°)0.9205
cos(338°)0.9272
cos(339°)0.9336
cos(340°)0.9397
cos(341°)0.9455
cos(342°)0.9511
cos(343°)0.9563
cos(344°)0.9613
cos(345°)0.9659
cos(346°)0.9703
cos(347°)0.9744
cos(348°)0.9781
cos(349°)0.9816
cos(350°)0.9848
cos(351°)0.9877
cos(352°)0.9903
cos(353°)0.9925
cos(354°)0.9945
cos(355°)0.9962
cos(356°)0.9976
cos(357°)0.9986
cos(358°)0.9994
cos(359°)0.9998
cos(360°)1

Как легко запомнить таблицу косинусов (видео)

Существуют также следующие таблицы тригонометрических функций: таблица синусов, таблица тангенсов и таблица котангенсов.

Всё для учебы » Математика в школе » Таблица косинусов (полная, градусы и значения)

Калейдоскоп формул для пи

Калейдоскоп формул для пи

«…я считал, что есть две математики — алгебраическая и геометрическая, и что геометрическая математика принципиально “трансцендентна” для алгебраической. Возьмите, например, формулу длины окружности — там есть “геометрическое” число $\pi$. Или, скажем, синус — он определяется чисто геометрически.

Когда я обнаружил, что синус можно записать алгебраически в виде ряда, барьер обрушился, математика стала единой.»

— из интервью И. М. Гельфанда

«Калейдоскоп» ниже состоит из нескольких «алгебраических» формул для $\pi$ с краткими комментариями. Он также опубликован (с сокращениями) в журнале «Квант» (№5 за 2020 год).

1. Формула Виета

Одна из первых алгебраических формул для $\pi$ — это открытое в XVI веке Виетом бесконечное произведение $$ \frac\pi2=\frac2{\sqrt2}\cdot\frac2{\sqrt{2+\sqrt2}}\cdot\frac2{\sqrt{2+\sqrt{2+\sqrt2}}}\cdot\ldots $$ Это равенство не очень сложно доказать.2$ (последнее равенство — это, по сути, основная теорема арифметики). Более серьезное обсуждение вопроса можно найти, например, в книге «Введение в теорию чисел» Харди и Райта.

4. Формула Валлиса

Если подставить $x=\pi/2$ в разложение Эйлера синуса в бесконечное произведение, то получается равенство $$ \frac\pi2= \frac{2\cdot2\cdot4\cdot4\cdot6\cdot6\cdot\ldots}{1\cdot3\cdot3\cdot5\cdot5\cdot7\cdot\ldots} $$ Впрочем, Джон Валлис нашел эту формулу уже в середине XVII века, почти за 100 лет до формулы Эйлера, вычисляя некоторые интегралы.

В упоминавшейся выше статье Ягломов при помощи элементарной тригонометрии доказывается и формула Валлиса. А J. Wästlund нашел и доказательство (в духе «геометрического суммирования»), непосредственно связывающее произведение Валлиса с площадью круга — см. его статью (AMM, 2007) или лекцию Д. Кнута.

При помощи формулы Валлиса можно доказать, что если подкинуть монету $2n$ раз, то вероятность того, что орлов и решек выпадет в точности поровну, приблизительно равна $1/\sqrt{\pi n}$.2+t-1=0, корни которого t_1=-1 и t_2=\frac12. Возвращаясь к переменной x, получим \cos x = \frac12 или \cos x=-1, откуда x=\frac \pi 3+2\pi m, m \in \mathbb Z, x=-\frac \pi 3+2\pi n, n \in \mathbb Z, x=\pi +2\pi k, k \in \mathbb Z.

б) Решим неравенства

1) -\frac{3\pi }2 \leqslant \frac{\pi }3+2\pi m \leqslant -\frac \pi 2 ,

2) -\frac{3\pi }2 \leqslant -\frac \pi 3+2\pi n \leqslant -\frac \pi {2,}

3) -\frac{3\pi }2 \leqslant \pi+2\pi k \leqslant -\frac \pi 2 , m, n, k \in \mathbb Z. 

Решение:

1) -\frac{3\pi }2 \leqslant \frac{\pi }3+2\pi m \leqslant -\frac \pi 2 , -\frac32 \leqslant  \frac13+2m \leqslant  -\frac12 -\frac{11}6 \leqslant  2m \leqslant  -\frac56 , -\frac{11}{12} \leqslant m \leqslant -\frac5{12}.

Нет целых чисел, принадлежащих промежутку \left [-\frac{11}{12};-\frac5{12}\right].

2) -\frac {3\pi} 2 \leqslant -\frac{\pi }3+2\pi n \leqslant -\frac{\pi }{2}, -\frac32 \leqslant -\frac13 +2n \leqslant -\frac12 , -\frac76 \leqslant 2n \leqslant -\frac1{6}, -\frac7{12} \leqslant n \leqslant -\frac1{12}.

Нет целых чисел, принадлежащих промежутку \left[ -\frac7{12} ; -\frac1{12} \right].

3) -\frac{3\pi }2 \leqslant \pi +2\pi k\leqslant -\frac{\pi }2, -\frac32 \leqslant 1+2k\leqslant -\frac12, -\frac52 \leqslant 2k \leqslant -\frac32, -\frac54 \leqslant k \leqslant -\frac34.

Этому неравенству удовлетворяет k=-1, тогда x=-\pi.

Триггерные значения на единичной окружности

Java-игры: карточки, сопоставление, концентрация и поиск слов.

Вы попрактикуетесь в нахождении триггерных значений углов на единичной окружности.

A B
sin pi / 6 1/2
cos pi / 6 sq root3 / 2
tan pi / 6 sq root3 / 3
sin pi / 4 sq root2 / 2
cos pi / 3 1/2
tan pi / 3 sqroot 3
cos pi / 2 0
sin pi / 2 1
cos 2pi / 3 -1/2
cos 3pi / 4 -sqroot2 / 2
cos 5pi / 6 -sqroot 3 / 2
cos pi / 4 sqroot2 / 2
tan pi / 4 1
sin pi / 3 sqroot3 / 2
tan pi / 2 undefined
sin 2pi / 3 sqroot3 / 2
tan 2pi / 3 с qroot 3
sin 3pi / 4 sqroot2 / 2
tan 3pi / 4-1
sin 5 pi / 6 1/2
tan 5pi / 6 -sqroot3 / 3
sin pi 0
cos pi-1
tan pi 0
sin 7pi / 6 -1/2
cos 7pi / 6 -sqroot3 / 2
tan 7pi / 6 sqroot 3/3
sin 5pi / 4 -sqroot2 / 2
cos 5pi / 4 — sqroot2 / 2
tan 5pi / 4 1
sin 4pi / 3 -sqroot3 / 2
cos 4pi / 3 -1/2
tan 4pi / 3 sqroot 3
sin 3pi / 2-1
cos 3pi / 2 0
tan 3pi / 2 undefined
sin 5pi / 3 -sqroot3 / 2
cos 5pi / 3 1/2
tan 5pi / 3 -sqroot3
sin 7pi / 4 -sqroot2 / 2
cos 7pi / 4 sqroot2 / 2
tan 7pi / 4-1
sin 11pi / 6 -1/2
cos 11pi / 6 sqroot3 / 2
tan 11pi / 6 -sqroot3 / 3
sin 2pi 0
cos 2pi 1
tan 2pi 0
sin 0pi 0
cos 0pi 1
tan 0pi 0

Что такое cos при Pi 2? — Мворганизация.org

Что такое cos у Пи 2?

cos (π2) = 0.

Не определен ли Cos Pi 2?

Значение cos (пи / 2) равно 0, поэтому секанс (пи) / 2 должен быть неопределенным.

Какое значение Cos дает 0?

Синусы и косинусы для специальных общих углов

Градусов Радианы косинус
60 ° π / 3 1/2
45 ° π / 4 √2 / 2
30 ° π / 6 √3 / 2
0 ° 1

Где Pi 2 на единичной окружности?

2 ответа.Интервал (−π2, π2) — правая половина единичной окружности.

Что находится между PI 2 и PI?

3π / 2 находится на полпути между π и 2π.

Число Пи 180 градусов?

Дело в том, что пи радиан равен 180 градусам. Радианы — это единица измерения углов, как и градусы, а пи — это просто количество радианов, составляющих этот угол. Так же, как один радиан равен 57,3 градусам (приблизительно).

Что такое угол Пи?

Радиан — это единица измерения углов, определяемых отношением длины дуги окружности к радиусу этой окружности.Один радиан — это угол, при котором это отношение равно единице (см. Первую диаграмму). 180 градусов = PI радиан, 360 градусов = 2 * PI радиан, 90 градусов = PI / 2 радиана и т. Д.

Сколько радианов в 120 градусах в Пи?

Ответы (для проверки вашей работы): (а) пи / 6 или 0,52 радиана.

Сколько радианов в 90 градусах в Пи?

Следовательно, из приведенного выше уравнения мы можем сказать, что 180 градусов равны π радиан… Градусы в радианы.

Угол в градусах Угол в радианах
60 ° π / 3 = 1.047 Рад
90 ° π / 2 = 1,571 рад
120 ° 2π / 3 = 2,094 рад
150 ° 5π / 6 = 2,618 рад

Сколько радианов в 270 градусах в Пи?

Итак, 1 = пи / 180, мы можем умножить любой градус на эту дробь, чтобы перейти от градусов к радианам. Например, 270 градусов умноженные на пи / 180, мы получим 270 пи / 180, что составляет 27/18. Разделите верхнюю и нижнюю части на 9, мы получим 3 пи / 2, так что 3 пи / 2 равно 270 градусам.Фактически, мы можем определить четыре квадранта в терминах или радианах.

Что такое 225 градусов в радианах в единицах Пи?

Конечно, есть. Угол в 225 градусов равен половине круга плюс 45 градусов. В терминах числа пи для углов в радианах это будет посередине между 1 пи и 1,5 пи, потому что 1 пи соответствует 180 градусам, а 1,5 пи соответствует 270 градусам. Итак, наш ответ 1,25 пи точно соответствует цели.

Сколько Пи больше 9 в градусах?

20∘

Сколько градусов в π 5 радианах?

Угловой стол

Градусов Радианы Двоичные радианы (штифт)
15 ° Pi / 12 2731
30 ° Pi / 6 5461
36 ° Pi / 5 6554
45 ° Pi / 4 8192

Какой угол пи 2?

Общие углы

Градусов Радианы
90 ° π / 2
60 ° π / 3
45 ° π / 4
30 ° π / 6

Сколько Пи делится на 2 в градусах?

Градусы и радианы

А B
30 градусов пи / 6 радиан
45 градусов пи / 4 радиана
60 градусов пи / 3 радиана
90 градусов пи / 2 радиана

Что такое градусы в радианах?

Измерение угла в радианах численно равно длине соответствующей дуги единичного круга.Один радиан равен 180 / π (~ 57,296) градусам. История / происхождение: Измерение углов по длине дуги использовалось математиками еще с 1400 года.

Почему мы переводим градусы в радианы?

Расчет всегда производится в радианах. Градус (прямой угол — 90 градусов) и градианная мера (прямой угол — 100 градусов) имеют свое применение. Радианы позволяют связать линейную меру и угловую меру. Единичный круг — это круг, радиус которого равен одной единице.

Что такое один радиан в Пи?

Радиан: Один радиан равен 180 градусам на пи.

Какая единица измерения 0 в радианах?

Таблица преобразования градусов в радианы

Градусы (°) Радианы (рад) Радианы (рад)
0 ° 0 рад 0 рад
30 ° π / 6 рад 0.рад
45 ° π / 4 рад 0.рад
60 ° π / 3 рад 1.рад

Могут ли радианы быть отрицательными?

1 Ответ. да. Как вы заметили, преобразование радианов в градусы — это просто умножение на положительную константу, поэтому они всегда будут иметь один и тот же знак. Но во многих случаях угол θ действует так же, как θ + 360∘, поэтому в этом случае положительная величина в радианах может действовать так же, как и отрицательная величина в градусах.

Как упростить радианы?

радиан в градусе:

  1. Укажите в пропорции радиан.
  2. Упростите комплексную дробь справа, умножив числитель на обратную величину знаменателя.
  3. Умножьте каждую сторону пропорции на 180.
  4. Уменьшите и упростите дробь справа. Итак, радианы эквивалентны 15 градусам.

Три доказательства триггерной идентичности

Hi Nadene,

Я вижу три способа подтвердить эту личность и способ, предложенный намек, на мой взгляд, худший из трех.Доказательство, предложенное по подсказке:

cos (s + t) = cos (s) cos (t) — sin (s) sin (t)
и sin (s + t) = sin (s) cos (t) + cos (s) sin (t)

Пусть s = x и t = y — pi / 2 и примените первое из приведенных выше выражений, чтобы получить

cos [x + (y-pi / 2)] = cos (x) cos (y-pi / 2) -sin (x) sin (y-pi / 2)

Теперь пусть s = y и t = pi / 2 и примените выражения выше для cos (s + t) и sin (s + t).Упростите, используя значения cos (-pi / 2) и sin (-pi / 2), и вы получите выражение, эквивалентное грешить (x + y).

Второе доказательство проще.

Запишите cos [x + (y-pi / 2)] как cos [(x + y) -pi / 2)] и используйте выражение для cos (s + t) с s = x + y и t = -pi / 2. Упростите, используя значения cos (-pi / 2) и грех (-pi / 2).

Третье доказательство, на мой взгляд, самое простое.

Постройте графики y = sin (t) и y = cos (t).

y = sin (t)

y = cos (t)

Чтобы получить график y = cos (t-pi / 2), вы сдвинете ось y на пи / 2 единицы. налево, чтобы получить

y = cos (t-pi / 2)

Но это график y = sin (t) и следовательно, для любого t cos (t-pi / 2) = sin (t).

Ура,
Пенни

10.4: Тригонометрические тождества — математика LibreTexts

В разделе \ ref {CircularFunctions} мы увидели полезность пифагорейских тождеств в теореме \ ref {pythids} вместе с частными и взаимными тождествами в теореме \ ref {recipquotid}. Эти тождества не только помогли нам вычислить значения круговых функций для углов, они также были полезны для упрощения выражений, включающих круговые функции.В этом разделе мы представляем несколько наборов идентификаторов, которые используются в этом курсе и за его пределами. Наш первый набор тождеств — это тождества «Четный / Нечетный». \ Footnote {Как упоминалось в конце раздела \ ref {TheUnitCircle}, свойства круговых функций, если их рассматривать как функции углов в радианной мере, сохраняются одинаково хорошо, если мы рассматривайте эти функции как функции действительных чисел. Неудивительно, что свойства четных / нечетных круговых функций названы так, потому что они определяют косинус и секанс как четные функции, в то время как остальные четыре круговые функции являются нечетными.(См. Раздел \ ref {GraphsofFunctions}.)}

Примечание: четные / нечетные идентификаторы

Для всех применимых углов \ (\ theta \):

  • \ (\ cos (- \ theta) = \ cos (\ theta) \)
  • \ (\ сек (- \ theta) = \ сек (\ theta) \)
  • \ (\ sin (- \ theta) = — \ sin (\ theta) \)
  • \ (\ csc (- \ theta) = — \ csc (\ theta) \)
  • \ (\ tan (- \ theta) = — \ tan (\ theta) \)
  • \ (\ детская кроватка (- \ theta) = — \ детская кроватка (\ theta) \)

В свете Факторного и Взаимного тождеств, теоремы \ ref {recipquotid}, достаточно показать \ (\ cos (- \ theta) = \ cos (\ theta) \) и \ (\ sin (- \ theta) = — \ грех (\ тета) \).Остальные четыре круговые функции могут быть выражены в терминах \ (\ cos (\ theta) \) и \ (\ sin (\ theta) \), поэтому доказательства их четных / нечетных тождеств оставлены в качестве упражнений. Рассмотрим угол \ (\ theta \), нанесенный в стандартном положении. Пусть \ (\ theta_ {o} \) будет углом, совпадающим с \ (\ theta \) с \ (0 \ leq \ theta_ {o} <2 \ pi \). (Мы можем построить угол \ (\ theta_ {o} \), вращая против часовой стрелки от положительной оси \ (x \) - к конечной стороне \ (\ theta \), как показано ниже.) Поскольку \ (\ theta \) и \ (\ theta_ {o} \) являются концевыми, \ (\ cos (\ theta) = \ cos (\ theta_ {o}) \) и \ (\ sin (\ theta) = \ sin (\ тета_ {о}) \).

Теперь рассмотрим углы \ (- \ theta \) и \ (- \ theta_ {o} \). Поскольку \ (\ theta \) совпадает с \ (\ theta_ {o} \), существует некоторое целое число \ (k \), так что \ (\ theta = \ theta_ {o} + 2 \ pi \ cdot k \) . Следовательно, \ (- \ theta = — \ theta_ {o} — 2 \ pi \ cdot k = — \ theta_ {o} + 2 \ pi \ cdot (-k) \). Поскольку \ (k \) является целым числом, то же самое и \ ((- k) \), что означает, что \ (- \ theta \) совпадает с \ (- \ theta_ {o} \). Следовательно, \ (\ cos (- \ theta) = \ cos (- \ theta_ {o}) \) и \ (\ sin (- \ theta) = \ sin (- \ theta_ {o}) \). Пусть \ (P \) и \ (Q \) обозначают точки на конечных сторонах \ (\ theta_ {o} \) и \ (- \ theta_ {o} \), соответственно, которые лежат на единичной окружности.По определению, координатами \ (P \) являются \ ((\ cos (\ theta_ {o}), \ sin (\ theta_ {o})) \), а координатами \ (Q \) являются \ (( \ cos (- \ theta_ {o}), \ sin (- \ theta_ {o})) \). Поскольку \ (\ theta_ {o} \) и \ (- \ theta_ {o} \) заметают конгруэнтные центральные сектора единичной окружности, отсюда следует, что точки \ (P \) и \ (Q \) симметричны относительно ось \ (x \). Таким образом, \ (\ cos (- \ theta_ {o}) = \ cos (\ theta_ {o}) \) и \ (\ sin (- \ theta_ {o}) = — \ sin (\ theta_ {o}) \). Поскольку косинусы и синусы \ (\ theta_ {o} \) и \ (- \ theta_ {o} \) такие же, как у \ (\ theta \) и \ (- \ theta \), соответственно, мы получить \ (\ cos (- \ theta) = \ cos (\ theta) \) и \ (\ sin (- \ theta) = — \ sin (\ theta) \), как требуется.Четные / нечетные тождества легко демонстрируются с использованием любого из «общих углов», указанных в Разделе \ ref {TheUnitCircle}. Однако их истинная полезность заключается не в вычислениях, а в упрощении выражений, включающих круговые функции. Фактически, наша следующая партия идентификаторов интенсивно использует четные / нечетные идентификаторы.

Примечание: тождества суммы и разности для косинуса

Для всех углов \ (\ alpha \) и \ (\ beta \):

  • \ (\ соз (\ альфа + \ бета) = \ соз (\ альфа) \ соз (\ бета) — \ грех (\ альфа) \ грех (\ бета) \)
  • \ (\ соз (\ альфа — \ бета) = \ соз (\ альфа) \ соз (\ бета) + \ грех (\ альфа) \ грех (\ бета) \)

Сначала докажем результат для разностей.Как и в доказательстве четных / нечетных тождеств, мы можем свести доказательство для общих углов \ (\ alpha \) и \ (\ beta \) к углам \ (\ alpha_ {o} \) и \ (\ beta_ {o } \), совмещенный с \ (\ alpha \) и \ (\ beta \), соответственно, каждый из которых имеет размер между \ (0 \) и \ (2 \ pi \) радианами. Поскольку \ (\ alpha \) и \ (\ alpha_ {o} \) на одном конце, как и \ (\ beta \) и \ (\ beta_ {o} \), отсюда следует, что \ (\ alpha — \ beta \ ) совпадает с \ (\ alpha_ {o} — \ beta_ {o} \). Рассмотрим нижеприведенный случай, когда \ (\ alpha_ {o} \ geq \ beta_ {o} \).

Поскольку углы \ (POQ \) и \ (AOB \) совпадают, расстояние между \ (P \) и \ (Q \) равно расстоянию между \ (A \) и \ (B \). \ footnote {На рисунке, который мы нарисовали, \ underline {tri} углы \ (POQ \) и \ (AOB \) совпадают, что даже лучше. Однако \ (\ alpha_ {o} — \ beta_ {o} \) может быть \ (0 \) или \ (\ pi \), ни один из которых не образует треугольник. Он также может быть больше, чем \ (\ pi \), что делает треугольник, но не тот, который мы нарисовали. Вы должны подумать об этих трех случаях.2 & = & 2 — 2 \ cos (\ alpha_ {o} — \ beta_ {o}) \\ \ end {array} \]

Собирая все вместе, получаем \ (2 — 2 \ cos (\ alpha_ {o}) \ cos (\ beta_ {o}) — 2 \ sin (\ alpha_ {o}) \ sin (\ beta_ {o} ) = 2 — 2 \ cos (\ alpha_ {o} — \ beta_ {o}) \), что упрощается до: \ (\ cos (\ alpha_ {o} — \ beta_ {o}) = \ cos (\ alpha_ {o}) \ cos (\ beta_ {o}) + \ sin (\ alpha_ {o}) \ sin (\ beta_ {o}) \). Поскольку \ (\ alpha \) и \ (\ alpha_ {o} \), \ (\ beta \) и \ (\ beta_ {o} \) и \ (\ alpha — \ beta \) и \ (\ alpha_ { o} — \ beta_ {o} \) все концевые пары углов, мы имеем \ (\ cos (\ alpha — \ beta) = \ cos (\ alpha) \ cos (\ beta) + \ sin (\ alpha) \ грех (\ бета) \).Для случая, когда \ (\ alpha_ {o} \ leq \ beta_ {o} \), мы можем применить приведенный выше аргумент к углу \ (\ beta_ {o} — \ alpha_ {o} \), чтобы получить тождество \ (\ cos (\ beta_ {o} — \ alpha_ {o}) = \ cos (\ beta_ {o}) \ cos (\ alpha_ {o}) + \ sin (\ beta_ {o}) \ sin (\ alpha_ {o}) \). Применяя четную идентичность косинуса, мы получаем \ (\ cos (\ beta_ {o} — \ alpha_ {o}) = \ cos (- (\ alpha_ {o} — \ beta_ {o})) = \ cos (\ alpha_ {o} — \ beta_ {o}) \), и в этом случае мы тоже получаем идентичность.

Чтобы получить идентичность суммы для косинуса, мы используем формулу разности вместе с идентичностями четности / нечетности

\ [\ cos (\ alpha + \ beta) = \ cos (\ alpha — (- \ beta)) = \ cos (\ alpha) \ cos (- \ beta) + \ sin (\ alpha) \ sin (- \ beta) = \ cos (\ alpha) \ cos (\ beta) — \ sin (\ alpha) \ sin (\ beta) \]

Мы нашли хорошее применение этим вновь обретенным идентификаторам в следующем примере.{\ circ} \ right) \\ & = & \ left (\ dfrac {\ sqrt {2}} {2} \ right) \ left (\ dfrac {\ sqrt {3}} {2} \ right) + \ left (\ dfrac {\ sqrt {2}} {2} \ right) \ left (\ dfrac {1} {2} \ right) \\ & = & \ dfrac {\ sqrt {6} + \ sqrt {2} } {4} \\ \ end {array} \]

  1. Прямым применением теоремы \ ref {cosinesumdifference} находим

\ [\ begin {array} {rcl} \ cos \ left (\ dfrac {\ pi} {2} — \ theta \ right) & = & \ cos \ left (\ dfrac {\ pi} {2} \ справа) \ cos \ left (\ theta \ right) + \ sin \ left (\ dfrac {\ pi} {2} \ right) \ sin \ left (\ theta \ right) \\ & = & \ left (0 \ справа) \ влево (\ cos (\ theta) \ right) + \ left (1 \ right) \ left (\ sin (\ theta) \ right) \\ & = & \ sin (\ theta) \\ \ end { массив} \]

Идентичность, подтвержденная в Примере \ (\ PageIndex {1} \), а именно: \ (\ cos \ left (\ frac {\ pi} {2} — \ theta \ right) = \ sin (\ theta) \), является первой из знаменитых идентичностей «совместной работы».Впервые на эти тождества намекали в упражнении \ ref {cofunctionforeshadowing} в разделе \ ref {TheUnitCircle}. Из \ (\ sin (\ theta) = \ cos \ left (\ frac {\ pi} {2} — \ theta \ right) \) получаем:

\ [\ sin \ left (\ dfrac {\ pi} {2} — \ theta \ right) = \ cos \ left (\ dfrac {\ pi} {2} — \ left [\ dfrac {\ pi} {2 } — \ theta \ right] \ right) = \ cos (\ theta), \]

, который на словах говорит, что синус угла является синусом его дополнения. Теперь, когда эти тождества были установлены для косинуса и синуса, остальные круговые функции следуют их примеру.Остальные доказательства оставим в качестве упражнений.

Примечание: идентификаторы совместных функций

Для всех применимых углов \ (\ theta \):

  • \ (\ cos \ left (\ dfrac {\ pi} {2} — \ theta \ right) = \ sin (\ theta) \)
  • \ (\ sin \ left (\ dfrac {\ pi} {2} — \ theta \ right) = \ cos (\ theta) \)
  • \ (\ sec \ left (\ dfrac {\ pi} {2} — \ theta \ right) = \ csc (\ theta) \)
  • \ (\ csc \ left (\ dfrac {\ pi} {2} — \ theta \ right) = \ sec (\ theta) \)
  • \ (\ tan \ left (\ dfrac {\ pi} {2} — \ theta \ right) = \ cot (\ theta) \)
  • \ (\ cot \ left (\ dfrac {\ pi} {2} — \ theta \ right) = \ tan (\ theta) \)

Имея идентификаторы совместных функций, мы теперь в состоянии вывести формулы суммы и разности для синуса.Чтобы вывести формулу суммы для синуса, мы преобразуем его в косинусы, используя тождество кофункции, а затем расширяем, используя формулу разности для косинуса

.

\ [\ begin {array} {rcl} \ sin (\ alpha + \ beta) & = & \ cos \ left (\ dfrac {\ pi} {2} — (\ alpha + \ beta) \ right) \\ & = & \ cos \ left (\ left [\ dfrac {\ pi} {2} — \ alpha \ right] — \ beta \ right) \\ & = & \ cos \ left (\ dfrac {\ pi} {2 } — \ alpha \ right) \ cos (\ beta) + \ sin \ left (\ dfrac {\ pi} {2} — \ alpha \ right) \ sin (\ beta) \\ & = & \ sin (\ alpha ) \ cos (\ beta) + \ cos (\ alpha) \ sin (\ beta) \\ \ end {array} \]

Мы можем вывести формулу разности для синуса, переписав \ (\ sin (\ alpha — \ beta) \) как \ (\ sin (\ alpha + (- \ beta)) \) и используя формулу суммы и четное / Странные личности.Опять же, подробности оставляем читателю.

Тождества суммы и разности для синуса

Для всех углов \ (\ alpha \) и \ (\ beta \), \ index {Идентичность различия! for sine} \ index {Сумма идентичности! для синуса}

  • \ (\ sin (\ alpha + \ beta) = \ sin (\ alpha) \ cos (\ beta) + \ cos (\ alpha) \ sin (\ beta) \)
  • \ (\ sin (\ alpha — \ beta) = \ sin (\ alpha) \ cos (\ beta) — \ cos (\ alpha) \ sin (\ beta) \)

Пример \ (\ PageIndex {1} \):

  1. Найдите точное значение \ (\ sin \ left (\ frac {19 \ pi} {12} \ right) \)
  2. Если \ (\ alpha \) — угол квадранта II с \ (\ sin (\ alpha) = \ frac {5} {13} \), а \ (\ beta \) — угол квадранта III с \ (\ tan (\ beta) = 2 \), найдите \ (\ sin (\ alpha — \ beta) \).
  3. Выведите формулу для \ (\ tan (\ alpha + \ beta) \) через \ (\ tan (\ alpha) \) и \ (\ tan (\ beta) \).

Раствор

  1. Как и в примере \ ref {cosinesumdiffex}, нам нужно записать угол \ (\ frac {19 \ pi} {12} \) как сумму или разность общих углов. Знаменатель \ (12 \) предполагает комбинацию углов со знаменателями \ (3 \) и \ (4 \). Одна из таких комбинаций — \ (\; \ frac {19 \ pi} {12} = \ frac {4 \ pi} {3} + \ frac {\ pi} {4} \). Применяя теорему \ ref {sinesumdifference}, получаем

\ [\ begin {array} {rcl} \ sin \ left (\ dfrac {19 \ pi} {12} \ right) & = & \ sin \ left (\ dfrac {4 \ pi} {3} + \ dfrac {\ pi} {4} \ right) \\ & = & \ sin \ left (\ dfrac {4 \ pi} {3} \ right) \ cos \ left (\ dfrac {\ pi} {4} \ right ) + \ cos \ left (\ dfrac {4 \ pi} {3} \ right) \ sin \ left (\ dfrac {\ pi} {4} \ right) \\ & = & \ left (- \ dfrac {\ sqrt {3}} {2} \ right) \ left (\ dfrac {\ sqrt {2}} {2} \ right) + \ left (- \ dfrac {1} {2} \ right) \ left (\ dfrac {\ sqrt {2}} {2} \ right) \\ & = & \ dfrac {- \ sqrt {6} — \ sqrt {2}} {4} \\ \ end {array} \]

  1. Чтобы найти \ (\ sin (\ alpha — \ beta) \) с помощью теоремы \ ref {sinesumdifference}, нам нужно найти \ (\ cos (\ alpha) \) и оба \ (\ cos (\ beta ) \) и \ (\ sin (\ beta) \).{2} (\ beta) = 1 \), но вместо этого мы предпочитаем использовать факторное тождество. Из \ (\ tan (\ beta) = \ frac {\ sin (\ beta)} {\ cos (\ beta)} \) имеем \ (\ sin (\ beta) = \ tan (\ beta) \ cos (\ beta) \), поэтому мы получаем \ (\ sin (\ beta) = (2) \ left (- \ frac {\ sqrt {5}} {5} \ right) = — \ frac {2 \ sqrt {5 }} {5} \). Теперь у нас есть все необходимое, чтобы найти \ (\ sin (\ alpha — \ beta) \):

\ [\ begin {array} {rcl} \ sin (\ alpha — \ beta) & = & \ sin (\ alpha) \ cos (\ beta) — \ cos (\ alpha) \ sin (\ beta) \ \ & = & \ left (\ dfrac {5} {13} \ right) \ left (- \ dfrac {\ sqrt {5}} {5} \ right) — \ left (- \ dfrac {12} {13} \ right) \ left (- \ dfrac {2 \ sqrt {5}} {5} \ right) \\ & = & — \ dfrac {29 \ sqrt {5}} {65} \\ \ end {array} \ ]

Мы можем начать расширение \ (\ tan (\ alpha + \ beta) \), используя частное тождество и наши формулы суммы

\ [\ begin {array} {rcl} \ tan (\ alpha + \ beta) & = & \ dfrac {\ sin (\ alpha + \ beta)} {\ cos (\ alpha + \ beta)} \\ & = & \ dfrac {\ sin (\ alpha) \ cos (\ beta) + \ cos (\ alpha) \ sin (\ beta)} {\ cos (\ alpha) \ cos (\ beta) — \ sin (\ alpha ) \ sin (\ beta)} \\ \ end {array} \]

Поскольку \ (\ tan (\ alpha) = \ frac {\ sin (\ alpha)} {\ cos (\ alpha)} \) и \ (\ tan (\ beta) = \ frac {\ sin (\ beta) } {\ cos (\ beta)} \), похоже, что если мы разделим числитель и знаменатель на \ (\ cos (\ alpha) \ cos (\ beta) \), мы получим то, что хотим

\ [\ begin {array} {rcl} \ tan (\ alpha + \ beta) & = & \ dfrac {\ sin (\ alpha) \ cos (\ beta) + \ cos (\ alpha) \ sin (\ beta )} {\ cos (\ alpha) \ cos (\ beta) — \ sin (\ alpha) \ sin (\ beta)} \ cdot \ dfrac {\ dfrac {1} {\ cos (\ alpha) \ cos (\ beta)}} {\ dfrac {1} {\ cos (\ alpha) \ cos (\ beta)}} \\ & & \\ & = & \ dfrac {\ dfrac {\ sin (\ alpha) \ cos (\ beta)} {\ cos (\ alpha) \ cos (\ beta)} + \ dfrac {\ cos (\ alpha) \ sin (\ beta)} {\ cos (\ alpha) \ cos (\ beta)}} { \ dfrac {\ cos (\ alpha) \ cos (\ beta)} {\ cos (\ alpha) \ cos (\ beta)} — \ dfrac {\ sin (\ alpha) \ sin (\ beta)} {\ cos (\ alpha) \ cos (\ beta)}} \\ & & \\ & = & \ dfrac {\ dfrac {\ sin (\ alpha) \ cancel {\ cos (\ beta)}} {\ cos (\ alpha ) \ cancel {\ cos (\ beta)}} + \ dfrac {\ cancel {\ cos (\ alpha)} \ sin (\ beta)} {\ cancel {\ cos (\ alpha)} \ cos (\ beta) }} {\ dfrac {\ cancel {\ cos (\ alpha)} \ cancel {\ cos (\ beta)}} {\ cancel {\ cos (\ alpha)} \ cancel {\ cos (\ beta)}} — \ dfrac {\ sin (\ alpha) \ sin (\ beta)} {\ cos (\ alpha) \ cos (\ beta)}} \\ & & \\ & = & \ dfrac {\ tan (\ alpha) + \ tan (\ beta)} {1 — \ tan (\ alpha) \ tan (\ beta)} \\ \ end {array} \]

Естественно, эта формула ограничена теми случаями, когда определены все касательные.\ qed

Формула, разработанная в упражнении \ ref {sinesumanddiffex} для \ (\ tan (\ alpha + \ beta) \), может использоваться, чтобы найти формулу для \ (\ tan (\ alpha — \ beta) \), переписав разницу в виде суммы \ (\ tan (\ alpha + (- \ beta)) \), и читателю предлагается заполнить детали. Ниже мы суммируем все формулы суммы и разности для косинуса, синуса и тангенса.

Примечание

Тождества суммы и разности:} Для всех применимых углов \ (\ alpha \) и \ (\ beta \), \ index {Идентичность разницы! для касательной} \ index {Sum Identity! по касательной} \ index {Отличие! для косинуса} \ index {Sum Identity! для косинуса} \ index {Отличие! for sine} \ index {Сумма идентичности! для синуса}

  • \ (\ cos (\ alpha \ pm \ beta) = \ cos (\ alpha) \ cos (\ beta) \ mp \ sin (\ alpha) \ sin (\ beta) \)
  • \ (\ sin (\ alpha \ pm \ beta) = \ sin (\ alpha) \ cos (\ beta) \ pm \ cos (\ alpha) \ sin (\ beta) \)
  • \ (\ tan (\ alpha \ pm \ beta) = \ dfrac {\ tan (\ alpha) \ pm \ tan (\ beta)} {1 \ mp \ tan (\ alpha) \ tan (\ beta)} \ )

В формулировке теоремы \ ref {circlesumdifference} мы объединили случаи для суммы `$ + $ ‘и разности углов` \) — $’ в одну формулу. {2} (\ theta) \ end {array} \ right.{2} (\ theta) = 1 \), а подробности оставляем читателю. Интересно отметить, что для определения значения \ (\ cos (2 \ theta) \) требуется только \ textit {one} часть информации: либо \ (\ cos (\ theta) \), либо \ (\ грех (\ тета) \). Однако, чтобы определить \ (\ sin (2 \ theta) \), мы должны знать как \ (\ sin (\ theta) \), так и \ (\ cos (\ theta) \). В следующем примере мы покажем, как найти \ (\ sin (2 \ theta) \), зная только одну часть информации, а именно \ (\ tan (\ theta) \).

Пример \ (\ PageIndex {1} \):

  1. Предположим, что \ (P (-3,4) \) лежит на конечной стороне \ (\ theta \), когда \ (\ theta \) отображается в стандартном положении.2 = — \ frac {7} {25} \) и \ (\ sin (2 \ theta) = 2 \ sin (\ theta) \ cos (\ theta) = 2 \ left (\ frac {4} {5 } \ right) \ left (- \ frac {3} {5} \ right) = — \ frac {24} {25} \). Поскольку и косинус, и синус \ (2 \ theta \) отрицательны, конечная сторона \ (2 \ theta \) при построении в стандартном положении находится в квадранте III. {3} (\ theta) — 3 \ cos (\ theta) \\ \ end {array} \]

    и все готово.{2} \ left (\ dfrac {\ theta} {2} \ right) = \ dfrac {1 + \ cos \ left (2 \ left (\ frac {\ theta} {2} \ right) \ right)} { 2} = \ dfrac {1 + \ cos (\ theta)} {2}. \]

    Мы можем получить формулу для \ (\ cos \ left (\ frac {\ theta} {2} \ right) \), извлекая квадратные корни. Аналогичным образом мы можем получить формулу половинного угла для синуса и, используя формулу частного, получить формулу половинного угла для тангенса. Мы резюмируем эти формулы ниже.

    Формулы полууглов

    Для всех применимых углов \ (\ theta \):

    • \ (\ cos \ left (\ dfrac {\ theta} {2} \ right) = \ pm \ sqrt {\ dfrac {1 + \ cos (\ theta)} {2}} \)
    • \ (\ sin \ left (\ dfrac {\ theta} {2} \ right) = \ pm \ sqrt {\ dfrac {1 — \ cos (\ theta)} {2}} \)
    • \ (\ tan \ left (\ dfrac {\ theta} {2} \ right) = \ pm \ sqrt {\ dfrac {1 — \ cos (\ theta)} {1+ \ cos (\ theta)}} \ )

    где выбор \ (\ pm \) зависит от квадранта, в котором лежит конечная сторона \ (\ dfrac {\ theta} {2} \).{\ circ} \ right) = \ frac {\ sqrt {6} + \ sqrt {2}} {4} \). Читателю предлагается доказать, что эти два выражения равны.

    1. Если \ (- \ pi \ leq \ theta \ leq 0 \), то \ (- \ frac {\ pi} {2} \ leq \ frac {\ theta} {2} \ leq 0 \), что означает \ (\ sin \ left (\ frac {\ theta} {2} \ right) <0 \). Теорема \ ref {halfangle} дает

    \ [\ begin {array} {rcl} \ sin \ left (\ dfrac {\ theta} {2} \ right) & = & — \ sqrt {\ dfrac {1- \ cos \ left (\ theta \ right )} {2}} = — \ sqrt {\ dfrac {1- \ left (- \ frac {3} {5} \ right)} {2}} \\ & = & — \ sqrt {\ dfrac {1 + \ frac {3} {5}} {2} \ cdot \ dfrac {5} {5}} = — \ sqrt {\ dfrac {8} {10}} = — \ dfrac {2 \ sqrt {5}} { 5} \\ \ end {array} \]

    1. Вместо нашего обычного подхода к проверке тождеств, а именно, начиная с одной стороны уравнения и пытаясь преобразовать ее в другую, мы начнем с тождества, которое мы доказали в номере \ ref {doubleanglesinewtan} примера \ ref {doubleangleex} и преобразовать его в личность, которую нас просят доказать.{2} \ left (\ frac {\ theta} {2} \ right) \\ \ sin (\ theta) & = & 2 \ tan \ left (\ frac {\ theta} {2} \ right) \ left ( \ dfrac {1 + \ cos \ left (2 \ left (\ frac {\ theta} {2} \ right) \ right)} {2} \ right) \\ \ sin (\ theta) & = & \ tan \ left (\ frac {\ theta} {2} \ right) \ left (1+ \ cos (\ theta) \ right) \\ \ tan \ left (\ dfrac {\ theta} {2} \ right) & = & \ dfrac {\ sin (\ theta)} {1+ \ cos (\ theta)} \\ \ end {array} \]

      Наша следующая партия идентичностей, Формулы суммирования продукта, \ footnote {Они также известны как формулы простафереза ​​и имеют богатую историю.Авторы рекомендуют вам провести над ними небольшое исследование, если это позволяет ваше расписание.} Легко проверить, развернув каждую из правых частей в соответствии с теоремой \ ref {circlesumdifference}, и, как вы уже могли ожидать, мы оставляем детали в качестве упражнений. Они особенно полезны в математическом анализе, и мы перечисляем их здесь для справки.

      Примечание: формулы произведения для суммирования

      Для всех углов \ (\ alpha \) и \ (\ beta \), \ index {Формулы произведения суммы}

      • \ (\ cos (\ alpha) \ cos (\ beta) = \ frac {1} {2} \ left [\ cos (\ alpha — \ beta) + \ cos (\ alpha + \ beta) \ right] \)
      • \ (\ sin (\ alpha) \ sin (\ beta) = \ frac {1} {2} \ left [\ cos (\ alpha — \ beta) — \ cos (\ alpha + \ beta) \ right] \ )
      • \ (\ sin (\ alpha) \ cos (\ beta) = \ frac {1} {2} \ left [\ sin (\ alpha — \ beta) + \ sin (\ alpha + \ beta) \ right] \ )

      Связанные с формулами произведения сумм относятся формулы суммы к продуктам, которые нам понадобятся в разделе \ ref {TrigEquIneq}.Их легко проверить с помощью формулы произведения на сумму, и поэтому их доказательства оставлены в качестве упражнений.

      Примечание: Сумма к формулам продукта:

      Для всех углов \ (\ alpha \) и \ (\ beta \):

      1. \ (\ cos (\ alpha) + \ cos (\ beta) = 2 \ cos \ left (\ dfrac {\ alpha + \ beta} {2} \ right) \ cos \ left (\ dfrac {\ alpha — \ beta} {2} \ right) \)
      2. \ (\ cos (\ alpha) — \ cos (\ beta) = — 2 \ sin \ left (\ dfrac {\ alpha + \ beta} {2} \ right) \ sin \ left (\ dfrac {\ alpha — \ beta} {2} \ right) \)
      3. \ (\ sin (\ alpha) \ pm \ sin (\ beta) = 2 \ sin \ left (\ dfrac {\ alpha \ pm \ beta} {2} \ right) \ cos \ left (\ dfrac {\ alpha \ mp \ beta} {2} \ right) \)

      Пример \ (\ PageIndex {1} \):

      1. Запишите \ (\; \ cos (2 \ theta) \ cos (6 \ theta) \; \) в виде суммы.
      2. \ Запишите \ (\; \ sin (\ theta) — \ sin (3 \ theta) \; \) как произведение.

      Раствор

      1. Идентифицируя \ (\ alpha = 2 \ theta \) и \ (\ beta = 6 \ theta \), мы находим

      \ [\ begin {array} {rcl} \ cos (2 \ theta) \ cos (6 \ theta) & = & \ frac {1} {2} \ left [\ cos (2 \ theta — 6 \ theta ) + \ cos (2 \ theta + 6 \ theta) \ right] \\ & = & \ frac {1} {2} \ cos (-4 \ theta) + \ frac {1} {2} \ cos (8 \ theta) \\ & = & \ frac {1} {2} \ cos (4 \ theta) + \ frac {1} {2} \ cos (8 \ theta), \ end {array} \]

      , где последнее равенство является результатом четного тождества косинуса, \ (\ cos (-4 \ theta) = \ cos (4 \ theta) \).

      1. Идентификация \ (\ alpha = \ theta \) и \ (\ beta = 3 \ theta \) дает

      \ [\ begin {array} {rcl} \ sin (\ theta) — \ sin (3 \ theta) & = & 2 \ sin \ left (\ dfrac {\ theta — 3 \ theta} {2} \ right ) \ cos \ left (\ dfrac {\ theta + 3 \ theta} {2} \ right) \\ & = & 2 \ sin \ left (- \ theta \ right) \ cos \ left (2 \ theta \ right) \\ & = & -2 \ sin \ left (\ theta \ right) \ cos \ left (2 \ theta \ right), \\ \ end {array} \]

      , где последнее равенство является результатом нечетного тождества синуса, \ (\ sin (- \ theta) = — \ sin (\ theta) \).

      Напоминаем читателю, что все тождества, представленные в этом разделе, которые рассматривают круговые функции как функции углов (в радианах), одинаково хорошо применимы к круговым (тригонометрическим) функциям, рассматриваемым как функции действительных чисел. В упражнениях \ ref {idengraphfirst} — \ ref {idengraphlast} в разделе \ ref {TrigGraphs} мы видим, как некоторые из этих тождеств проявляются геометрически, когда мы изучаем графики этих функций. Однако в следующих упражнениях вам нужно будет выполнять всю свою работу аналитически без графиков.

      Авторы и авторство

      • Карл Ститц, доктор философии (Lakeland Community College) и Джефф Зигер, доктор философии. (Общественный колледж округа Лорейн)

      Расширенный круг единиц

      Четкое понимание единичного круга делает тригонометрию намного проще понять. Вы должны абсолютно понимать, что Восток, Север, Запад и Юг точки (показаны зеленым цветом ниже) имеют угловые меры, которые кратные $ \ pi / 2 $ и их координаты равны $$ (1,0), \; (0,1) \; (-1,0) \; \ текст {и} \; (0, -1).$$ Это позволяет очень легко считывать значения синуса и косинуса. Триггерные значения $ \ pi / 6 $ и $ \ pi / 3 $ могут быть получены из равностороннего треугольник и триггерные значения $ \ pi / 4 $ могут быть получены из равнобедренного прямоугольный треугольник. Опять же, понимание единичного круга поможет вам быстро см. триггерные значения соответствующих углов, например $ -7 \ pi / 6 $.

      Вы можете использовать интерактивную версию единичного круга для проверки значений стандартные углы между нулем и $ 2 \ pi $. Вы также можете нажать «Показать больше углов». кнопку, если вас интересуют более безумные ракурсы.

      Показать больше ракурсов:

      Нахождение значений под более безумными углами

      Все значения, показанные в демонстрации выше, были вычислены с использованием системы Mathematica. Команда FunctionExpand . Вы можете сделать это с помощью больше углов. Например, вы можете вычислить $ \ cos (11 \ pi / 24) $ так:

      FunctionExpand [Cos [11 Pi / 24]]
       
      $ \ displaystyle \ frac {1} {4} \ sqrt {2- \ sqrt {2}} \ left (1+ \ sqrt {2} \ right) — \ frac {1} {4} \ left (\ sqrt {2} -1 \ right) \ sqrt {3 \ left (2+ \ sqrt {2} \ right)} $

      Вы даже можете передать результат в TeXForm в сгенерируйте команды TeX, необходимые для набора выражения.п = \ соз (\ пи) + я \ грех (\ пи) = — 1. $$

      Таким образом, если мы расширим член слева, установим действительную часть равной $ -1 $. n]; subbed = expr /.{2/3} \ right)} $

      Тригонометрические функции и единичная окружность

      Радианы

      Радианы — это еще один способ измерения углов, и величина угла может быть преобразована между градусами и радианами.

      Цели обучения

      Объясните определение радианов с точки зрения длины дуги единичного круга и используйте это для преобразования между градусами и радианами.

      Ключевые выносы

      Ключевые моменты
      • Один радиан — это мера центрального угла окружности, при которой длина дуги равна радиусу
        окружности.\ circ} {\ pi}} [/ латекс].
      • Радианная мера угла — это отношение длины дуги к радиусу круга [латекс] \ displaystyle {\ left (\ theta = \ frac {s} {r} \ right)} [/ latex] . Другими словами, если [latex] s [/ latex] — длина дуги круга, а [latex] r [/ latex] — радиус круга, то центральный угол, содержащий эту дугу, измеряется в радианах.
      Ключевые термины
      • дуга : Непрерывная часть окружности круга.
      • окружность : длина линии, ограничивающей круг.
      • радиан : Стандартная единица измерения углов в математике. Мера центрального угла круга, который пересекает дугу, равную по длине радиусу этого круга.

      Введение в радианы

      Напомним, что деление круга на 360 частей дает измерение в градусах. Это произвольное измерение, и мы можем выбрать другие способы разделить круг.Чтобы найти другую единицу, представьте себе процесс рисования круга. Представьте, что вы остановились до того, как круг замкнулся. Нарисованная вами часть называется дугой. Дуга может быть частью полного круга, полного круга или более чем полного круга, представленного более чем одним полным оборотом. Длина дуги вокруг всего круга называется окружностью этого круга.

      Окружность круга

      [латекс] C = 2 \ pi r [/ латекс]

      Если мы разделим обе части этого уравнения на [латекс] r [/ латекс], мы получим отношение длины окружности, которое всегда равно [латексу] 2 \ pi [/ латексу] к радиусу, независимо от длины радиус.Таким образом, длина окружности любого круга равна [латексу] 2 \ пи \ приблизительно в 6,28 [/ латексу] раз больше длины радиуса. Это означает, что если мы возьмем струну такой же длины, как радиус, и будем использовать ее для измерения последовательных длин по окружности, то будет место для шести полных струн и чуть больше четверти седьмой, как показано на диаграмме. ниже.

      Длина окружности по сравнению с радиусом : длина окружности чуть более чем в 6 раз превышает длину радиуса.

      Это подводит нас к нашей новой угловой мере. Радиан — это стандартная единица измерения углов в математике. Один радиан — это мера центрального угла круга, который пересекает дугу, равную по длине радиусу этого круга.

      Один радиан: Угол [латекс] t [/ латекс] выметает величину в один радиан. Обратите внимание, что длина перехваченной дуги равна длине радиуса круга.

      Поскольку общая длина окружности равна [латексу] 2 \ pi [/ latex], умноженному на радиус, полный круговой поворот составляет [латекс] 2 \ pi [/ latex] радиан. {\ circ}} [/ latex].

      Измерение угла в радианах

      Длина дуги [латекс] s [/ латекс] — это длина кривой вдоль дуги. Так же, как полная длина окружности всегда имеет постоянное отношение к радиусу, длина дуги, образованная любым заданным углом, также имеет постоянную связь с радиусом, независимо от длины радиуса.

      Это соотношение, называемое радианной мерой, одинаково независимо от радиуса круга — оно зависит только от угла. Это свойство позволяет нам определять меру любого угла как отношение длины дуги [latex] s [/ latex] к радиусу [latex] r [/ latex].

      [латекс] \ displaystyle {\ begin {align} s & = r \ theta \\ \ theta & = \ frac {s} {r} \ end {align}} [/ latex]

      Измерительные радианы: (a) Под углом 1 радиан; длина дуги равна радиусу [латекса] r [/ латекса]. (b) Угол в 2 радиана имеет длину дуги [латекс] s = 2r [/ латекс]. (c) Полный оборот составляет [латекс] 2 \ pi [/ латекс], или около 6,28 радиана.

      Пример

      Какова мера данного угла в радианах, если длина его дуги составляет [латекс] 4 \ pi [/ латекс], а радиус имеет длину [латекс] [/ латекс] 12?

      Подставьте значения [латекс] s = 4 \ pi [/ latex] и [latex] r = 12 [/ latex] в формулу угла:

      [латекс] \ displaystyle {\ begin {align} \ theta & = \ frac {s} {r} \\ & = \ frac {4 \ pi} {12} \\ & = \ frac {\ pi} {3 } \\ & = \ frac {1} {3} \ pi \ end {align}} [/ latex]

      Угол имеет размер [латекс] \ displaystyle {\ frac {1} {3} \ pi} [/ latex] радиан.

      Определение тригонометрических функций на единичной окружности

      Определение точек на единичной окружности позволяет применять тригонометрические функции к любому углу.

      Цели обучения

      Используйте прямоугольные треугольники, нарисованные в единичной окружности, чтобы определить тригонометрические функции для любого угла

      Ключевые выносы

      Ключевые моменты
      • Координаты [latex] x [/ latex] — и [latex] y [/ latex] в точке единичной окружности, заданной углом [latex] t [/ latex], определяются функциями [latex] x = \ cos t [/ latex] и [latex] y = \ sin t [/ latex].{\ circ} [/ латекс].
      • Единичный круг демонстрирует периодичность тригонометрических функций, показывая, что они приводят к повторяющемуся набору значений через равные промежутки времени.
      Ключевые термины
      • периодичность : качество функции с повторяющимся набором значений через равные промежутки времени.
      • единичная окружность : окружность с центром в начале координат и радиусом 1.
      • квадранты : Четыре четверти координатной плоскости, образованные осями [латекс] х [/ латекс] — и [латекс] у [/ латекс].

      Тригонометрические функции и единичная окружность

      Мы уже определили тригонометрические функции в терминах прямоугольных треугольников. В этом разделе мы переопределим их в терминах единичной окружности. Напомним, что единичный круг — это круг с центром в начале координат и радиусом 1. Угол [латекс] t [/ латекс] (в радианах) образует дугу длиной [латекс] s [/ латекс].

      Оси x- и y- делят координатную плоскость (и единичную окружность, поскольку она центрирована в начале координат) на четыре четверти, называемых квадрантами.Мы помечаем эти квадранты, чтобы имитировать направление, в котором развернется положительный угол. Четыре квадранта обозначены I, II, III и IV.

      Для любого угла [латекс] t [/ латекс] мы можем обозначить пересечение его стороны и единичного круга его координатами, [латекс] (x, y) [/ latex]. Координаты [latex] x [/ latex] и [latex] y [/ latex] будут выходными данными тригонометрических функций [latex] f (t) = \ cos t [/ latex] и [latex] f (t). = \ sin t [/ latex] соответственно. Это означает:

      [латекс] \ displaystyle {\ begin {align} x & = \ cos t \\ y & = \ sin t \ end {align}} [/ latex]

      Эти координаты показаны на диаграмме единичного круга.

      Единичный круг: Координаты точки на единичной окружности, центральный угол которой составляет [латекс] t [/ латекс] радиан.

      Обратите внимание, что значения [latex] x [/ latex] и [latex] y [/ latex] задаются длинами двух сторон треугольника, окрашенных в красный цвет. Это прямоугольный треугольник, и вы можете видеть, как длины этих двух сторон (и значения [latex] x [/ latex] и [latex] y [/ latex]) задаются тригонометрическими функциями [latex] t [/латекс].

      В качестве примера того, как это применимо, рассмотрим диаграмму, показывающую точку с координатами [latex] \ displaystyle {\ left (- \ frac {\ sqrt2} {2}, \ frac {\ sqrt2} {2} \ right)} [/ latex] по единичной окружности.

      Точка на единичном круге: точка [латекс] \ displaystyle {\ left (- \ frac {\ sqrt2} {2}, \ frac {\ sqrt2} {2} \ right)} [/ latex] на единичном круге .

      Мы знаем, что для любой точки единичного круга координата [latex] x [/ latex] равна [latex] \ cos t [/ latex], а координата [latex] y [/ latex] — [latex] ] \ sin t [/ латекс]. Применяя это, мы можем определить, что [latex] \ displaystyle {\ cos t = — \ frac {\ sqrt2} {2}} [/ latex] и [latex] \ displaystyle {\ sin t = — \ frac {\ sqrt2} {2}} [/ латекс] для угла [латекс] t [/ латекс] на схеме.

      Напомним, что [латекс] \ displaystyle {\ tan t = \ frac {\ sin t} {\ cos t}} [/ latex]. Применяя эту формулу, мы можем найти тангенс любого угла, обозначенного единичной окружностью. Для угла [латекс] t [/ латекс], указанного на диаграмме единичного круга, показывающего точку [латекс] \ displaystyle {\ left (- \ frac {\ sqrt2} {2}, \ frac {\ sqrt2} {2 } \ right)} [/ latex], касательная:

      [латекс] \ displaystyle {\ begin {align} \ tan t & = \ frac {\ sin t} {\ cos t} \\ & = \ frac {- \ frac {\ sqrt2} {2}} {- \ гидроразрыв {\ sqrt2} {2}} \\ & = 1 \ end {align}} [/ latex]

      Ранее мы обсуждали тригонометрические функции применительно к прямоугольным треугольникам.{\ circ} [/ латекс].

      Дальнейшее рассмотрение единичного круга

      Координаты определенных точек на единичной окружности и мера каждого угла в радианах и градусах показаны на диаграмме координат единичной окружности. Эта диаграмма позволяет наблюдать за каждым из этих углов, используя тригонометрические функции.

      Координаты единичной окружности : Единичная окружность, показывающая координаты и угловые размеры определенных точек.

      Мы можем найти координаты любой точки единичной окружности.Учитывая любой угол [латекс] t [/ латекс], мы можем найти координату [latex] x [/ latex] или [latex] y [/ latex] в этой точке, используя [latex] x = \ text {cos} t [/ latex] и [latex] y = \ text {sin} t [/ latex].

      Единичный круг демонстрирует периодичность тригонометрических функций. Периодичность относится к способу, которым тригонометрические функции приводят к повторяющемуся набору значений через равные промежутки времени. Взгляните на [latex] x [/ latex] -значения координат в единичном круге выше для значений [latex] t [/ latex] от [latex] 0 [/ latex] до [latex] 2 {\ pi} [/ latex]:

      [латекс] {1, \ frac {\ sqrt {3}} {2}, \ frac {\ sqrt {2}} {2}, \ frac {1} {2}, 0, — \ frac {1} {2}, — \ frac {\ sqrt {2}} {2}, — \ frac {\ sqrt {3}} {2}, -1, — \ frac {\ sqrt {3}} {2}, — \ frac {\ sqrt {2}} {2}, — \ frac {1} {2}, 0, \ frac {1} {2}, \ frac {\ sqrt {2}} {2}, \ frac { \ sqrt {3}} {2}, 1} [/ латекс]

      Мы можем определить закономерность в этих числах, которые колеблются между [латекс] -1 [/ латекс] и [латекс] 1 [/ латекс].Обратите внимание, что этот шаблон будет повторяться для более высоких значений [latex] t [/ latex]. Напомним, что эти значения [latex] x [/ latex] соответствуют [latex] \ cos t [/ latex]. Это показатель периодичности функции косинуса.

      Пример

      Решите [латекс] \ displaystyle {\ sin {\ left (\ frac {7 \ pi} {6} \ right)}} [/ latex].

      Похоже, это будет сложно решить. Однако обратите внимание, что диаграмма единичного круга показывает координаты в [latex] \ displaystyle {t = \ frac {7 \ pi} {6}} [/ latex].Поскольку координата [latex] y [/ latex] соответствует [latex] \ sin t [/ latex], мы можем идентифицировать, что

      [латекс] \ displaystyle {\ sin {\ left (\ frac {7 \ pi} {6} \ right)} = — \ frac {1} {2}} [/ latex]

      Специальные уголки

      Единичный круг и набор правил можно использовать для вызова значений тригонометрических функций специальных углов.

      Цели обучения

      Объясните, как свойства синуса, косинуса и тангенса и их знаки в каждом квадранте дают свои значения для каждого из специальных углов

      Ключевые выносы

      Ключевые моменты
      • Тригонометрические функции для углов в единичной окружности можно запомнить и вызвать с помощью набора правил.
      • Знак тригонометрической функции зависит от квадранта, в который попадает угол, и мнемоническая фраза «Умный класс триггера» используется для определения того, какие функции в каком квадранте положительны.
      • Базовые углы в квадранте I используются для определения значения любого угла в квадрантах II, III или IV. Базовый угол образует тот же угол с осью [latex] x [/ latex], что и рассматриваемый угол.
      • В единичную окружность включаются только функции синуса и косинуса для особых углов.Однако, поскольку тангенс получается из синуса и косинуса, его можно вычислить для любого из специальных углов.
      Ключевые термины
      • специальный угол : угол, кратный 30 или 45 градусам; тригонометрические функции легко записываются под этими углами.
      Тригонометрические функции специальных углов

      Напомним, что определенные углы и их координаты, которые соответствуют [latex] x = \ cos t [/ latex] и [latex] y = \ sin t [/ latex] для данного угла [latex] t [/ latex], можно идентифицировать на единичном круге.{\ circ} \ right)} & = 1 \\ \ end {align}} [/ latex]

      Выражения для косинусных функций этих специальных углов также просты.

      Обратите внимание, что, хотя только синус и косинус определяются непосредственно единичной окружностью, касательную можно определить как частное, включающее эти два:

      [латекс] \ displaystyle {\ tan t = \ frac {\ sin t} {\ cos t}} [/ латекс]

      Функции касания также имеют простые выражения для каждого из специальных углов.

      Мы можем наблюдать эту тенденцию на примере.{\ circ} \ right)}} \\ & = \ frac {\ frac {\ sqrt {3}} {2}} {\ frac {1} {2}} \\ & = \ frac {\ sqrt {3 }} {2} \ cdot \ frac {2} {1} \\ & = \ sqrt {3} \ end {align}} [/ latex]

      Запоминание тригонометрических функций

      Понимание единичной окружности и способность быстро решать тригонометрические функции для определенных углов очень полезно в области математики. Применение правил и ярлыков, связанных с единичным кругом, позволяет быстро решать тригонометрические функции. Ниже приведены некоторые правила, которые помогут вам быстро решить такие проблемы.

      Признаки тригонометрических функций

      Знак тригонометрической функции зависит от квадранта, в который попадает угол. Чтобы помочь запомнить, какие из тригонометрических функций положительны в каждом квадранте, мы можем использовать мнемоническую фразу «Умный класс триггера». Каждое из четырех слов во фразе соответствует одному из четырех квадрантов, начиная с квадранта I и вращаясь против часовой стрелки. В квадранте I, который является «А», все тригонометрические функции положительны.В квадранте II «Умный» только синус является положительным. В квадранте III «Триггер» только , касательная положительна. Наконец, в квадранте IV «Класс» только косинус является положительным.

      Правила знаков для тригонометрических функций: Каждая тригонометрическая функция перечислена в тех квадрантах, в которых она положительна.

      Определение значений с помощью опорных углов

      Внимательно посмотрите на единичный круг и обратите внимание, что [latex] \ sin t [/ latex] и [latex] \ cos t [/ latex] принимают определенные значения, поскольку они колеблются между [latex] -1 [/ latex] и [латекс] 1 [/ латекс]. {\ circ} [/ латекс].

      Для любого заданного угла в первом квадранте существует угол во втором квадранте с таким же значением синуса. Поскольку значение синуса является координатой [latex] y [/ latex] на единичной окружности, другой угол с таким же синусом будет иметь такое же значение [latex] y [/ latex], но будет иметь противоположное значение [latex] x [/ latex] -значение. Следовательно, его значение косинуса будет противоположным значению косинуса первого угла.

      Аналогично, в четвертом квадранте будет угол с таким же косинусом, что и исходный угол.Угол с таким же косинусом будет иметь одно и то же значение [latex] x [/ latex], но будет иметь противоположное значение [latex] y [/ latex]. Следовательно, его значение синуса будет противоположным значению синуса исходного угла.

      Как показано на диаграммах ниже, угол [латекс] \ альфа [/ латекс] имеет то же значение синуса, что и угол [латекс] t [/ латекс]; значения косинуса противоположны. Угол [латекс] \ бета [/ латекс] имеет то же значение косинуса, что и угол [латекс] t [/ латекс]; значения синуса противоположны.

      [латекс] \ Displaystyle {\ begin {align} \ sin t = \ sin \ alpha \ quad & \ text {and} \ quad \ cos t = — \ cos \ alpha \\ \ sin t = — \ sin \ beta \ quad & \ text {and} \ quad \ cos t = \ cos \ beta \ end {align}} [/ latex]

      Контрольные углы: На левом рисунке [латекс] t [/ latex] является контрольным углом для [латекс] \ альфа [/ латекс].{\ circ} [/ latex] или [latex] 0 [/ latex] и [latex] \ displaystyle {\ frac {\ pi} {2}} [/ latex] радиан. Для любого угла в квадранте II, III или IV существует опорный угол в квадранте I.

      Контрольные углы в каждом квадранте: Для любого угла в квадрантах II, III или IV существует контрольный угол в квадранте I.

      Таким образом, чтобы вызвать любой синус или косинус особого угла, вы должны иметь возможность идентифицировать его угол с осью [latex] x [/ latex], чтобы сравнить его с опорным углом.{\ circ})} \\ & = \ frac {- \ frac {\ sqrt {2}} {2}} {- \ frac {\ sqrt {2}} {2}} \\ & = — \ frac { \ sqrt {2}} {2} \ cdot — \ frac {2} {\ sqrt {2}} \\ & = 1 \ end {align}} [/ latex]

      Синус и косинус как функции

      Функции синуса и косинуса можно изобразить, используя значения из единичной окружности, и на обоих графиках можно наблюдать определенные характеристики.

      Цели обучения

      Опишите характеристики графиков синуса и косинуса

      Ключевые выносы

      Ключевые моменты
      • И синусоидальную функцию [latex] (y = \ sin x) [/ latex], и косинусную функцию [latex] (y = \ cos x) [/ latex] можно изобразить, нанеся точки, полученные из единичной окружности, с каждая координата [latex] x [/ latex] представляет собой угол в радианах, а координата [latex] y [/ latex] представляет собой соответствующее значение функции под этим углом.
      • Синус и косинус — периодические функции с периодом [латекс] 2 \ pi [/ латекс].
      • И синус, и косинус имеют домен [latex] (- \ infty, \ infty) [/ latex] и диапазон [latex] [- 1, 1] [/ latex].
      • График [latex] y = \ sin x [/ latex] симметричен относительно начала координат, потому что это нечетная функция, в то время как график [latex] y = \ cos x [/ latex] симметричен относительно [latex ] y [/ latex] -axis, потому что это четная функция.
      Ключевые термины
      • период : интервал, содержащий значения, повторяющиеся в функции.
      • четная функция : Непрерывный набор точек [latex] \ left (x, f (x) \ right) [/ latex], в которых [latex] f (-x) = f (x) [/ latex], с симметрией относительно оси [латекс] y [/ латекс].
      • нечетная функция : Непрерывный набор точек [latex] \ left (x, f (x) \ right) [/ latex], в которых [latex] f (-x) = -f (x) [/ latex] , с симметрией относительно начала координат.
      • периодическая функция : непрерывный набор точек [latex] \ left (x, f (x) \ right) [/ latex], повторяющихся через равные промежутки времени.

      Графические функции синуса и косинуса

      Напомним, что функции синуса и косинуса связывают значения действительных чисел с координатами [latex] x [/ latex] и [latex] y [/ latex] точки на единичной окружности. Так как же они выглядят на графике на координатной плоскости? Начнем с синусоидальной функции [latex] y = \ sin x [/ latex]. Мы можем создать таблицу значений и использовать их для построения графика. Ниже приведены некоторые значения для функции синуса на единичном круге, где координата [latex] x [/ latex] представляет собой угол в радианах, а координата [latex] y [/ latex] — [latex] \ sin х [/ латекс]:

      [латекс] \ displaystyle {(0, 0) \ quad (\ frac {\ pi} {6}, \ frac {1} {2}) \ quad (\ frac {\ pi} {4}, \ frac { \ sqrt {2}} {2}) \ quad (\ frac {\ pi} {3}, \ frac {\ sqrt {3}} {2}) \ quad (\ frac {\ pi} {2}, 1 ) \\ (\ frac {2 \ pi} {3}, \ frac {\ sqrt {3}} {2}) \ quad (\ frac {3 \ pi} {4}, \ frac {\ sqrt {2} } {2}) \ quad (\ frac {5 \ pi} {6}, \ frac {1} {2}) \ quad (\ pi, 0)} [/ latex]

      Построение точек из таблицы и продолжение по оси [latex] x [/ latex] дает форму синусоидальной функции.

      График синусоидальной функции: График точек с координатами [latex] x [/ latex], являющимися углами в радианах, и координатами [latex] y [/ latex], являющимися функцией [latex] \ sin x [/ latex] .

      Обратите внимание на положительные значения синуса между [latex] 0 [/ latex] и [latex] \ pi [/ latex], которые соответствуют значениям синусоидальной функции в квадрантах I и II на единичной окружности, и синусоидальной значения отрицательны между [латекс] \ пи [/ латекс] и [латекс] 2 \ пи [/ латекс], которые соответствуют значениям синусоидальной функции в квадрантах III и IV на единичной окружности.

      Построение значений синусоидальной функции: Точки на кривой [латекс] y = \ sin x [/ latex] соответствуют значениям синусоидальной функции на единичной окружности.

      Теперь давайте посмотрим на функцию косинуса, [latex] f (x) = \ sin x [/ latex]. Опять же, мы можем создать таблицу значений и использовать их для построения графика. Ниже приведены некоторые значения синусоидальной функции на единичном круге, где координата [latex] x [/ latex] представляет собой угол в радианах, а координата [latex] y [/ latex] — [latex] \ cos х [/ латекс]:

      [латекс] \ displaystyle {(0, 1) \ quad (\ frac {\ pi} {6}, \ frac {\ sqrt {3}} {2}) \ quad (\ frac {\ pi} {4} , \ frac {\ sqrt {2}} {2}) \ quad (\ frac {\ pi} {3}, \ frac {1} {2}) \ quad (\ frac {\ pi} {2}, 0 ) \\ (\ frac {2 \ pi} {3}, — \ frac {1} {2}) \ quad (\ frac {3 \ pi} {4}, — \ frac {\ sqrt {2}} { 2}) \ quad (\ frac {5 \ pi} {6}, — \ frac {\ sqrt {3}} {2}) \ quad (\ pi, -1)} [/ latex]

      Как и в случае с функцией синуса, мы можем построить точки для построения графика функции косинуса.

      График функции косинуса: Точки на кривой [latex] y = \ cos x [/ latex] соответствуют значениям функции косинуса на единичной окружности.

      Поскольку мы можем вычислять синус и косинус любого действительного числа, обе эти функции определены для всех действительных чисел. Если рассматривать значения синуса и косинуса как координаты точек на единичном круге, становится ясно, что диапазон обеих функций должен быть интервалом [латекс] \ left [-1, 1 \ right] [/ latex].

      Определение периодических функций

      На графиках для функций синуса и косинуса форма графика повторяется после [latex] 2 \ pi [/ latex], что означает, что функции являются периодическими с периодом [latex] 2 \ pi [/ latex].Периодическая функция — это функция с повторяющимся набором значений через равные промежутки времени. В частности, это функция, для которой конкретный сдвиг по горизонтали, [latex] P [/ latex], приводит к функции, равной исходной функции:

      [латекс] f (x + P) = f (x) [/ латекс]

      для всех значений [latex] x [/ latex] в домене [latex] f [/ latex]. Когда это происходит, мы называем наименьший такой сдвиг по горизонтали с [latex] P> 0 [/ latex] периодом функции. На приведенной ниже диаграмме показаны несколько периодов функций синуса и косинуса.

      Периоды функций синуса и косинуса: Функции синуса и косинуса являются периодическими, что означает, что определенный горизонтальный сдвиг, [latex] P [/ latex], приводит к функции, равной исходной функции: [latex] f (x + P) = f (x) [/ латекс].

      Четные и нечетные функции

      Еще раз взглянув на функции синуса и косинуса в домене с центром на оси [latex] y [/ latex], можно выявить симметрии. Как мы можем видеть на графике синусоидальной функции, она симметрична относительно начала координат, что указывает на то, что это нечетная функция.На всем протяжении графика любые две точки с противоположными значениями [latex] x [/ latex] также имеют противоположные значения [latex] y [/ latex]. Это характерно для нечетной функции: два входа, которые являются противоположными, имеют выходы, которые также являются противоположными. Другими словами, если [латекс] \ sin (-x) = — \ sin x [/ latex].

      Нечетная симметрия синусоидальной функции: Синусоидальная функция нечетная, что означает, что она симметрична относительно начала координат.

      График функции косинуса показывает, что он симметричен относительно оси y .Это указывает на то, что это четная функция. Для четных функций любые две точки с противоположными значениями [latex] x [/ latex] имеют одинаковое значение функции. Другими словами, [латекс] \ cos (-x) = \ cos x [/ latex]. Мы можем видеть из графика, что это правда, сравнивая значения [latex] y [/ latex] графика с любыми противоположными значениями [latex] x [/ latex].

      Четная симметрия функции косинуса: Функция косинуса четная, что означает, что она симметрична относительно оси [latex] y [/ latex].

      Касательная как функция

      Характеристики касательной функции можно увидеть на ее графике.

      Цели обучения

      Опишите характеристики графика касательной функции

      Ключевые выносы

      Ключевые моменты
      • Функция тангенса не определена при любом значении [latex] x [/ latex], где [latex] \ cos x = 0 [/ latex], и ее график имеет вертикальные асимптоты при этих значениях [latex] x [/ latex] .
      • Касательная — периодическая функция с периодом [латекс] \ пи [/ латекс].
      • График функции касательной симметричен относительно начала координат и, следовательно, является нечетной функцией.
      Ключевые термины
      • периодическая функция : непрерывный набор точек [latex] \ left (x, f (x) \ right) [/ latex] с набором значений, повторяющихся через равные промежутки времени.
      • период : интервал, содержащий минимальный набор значений, которые повторяются в периодической функции.
      • нечетная функция : Непрерывный набор точек [latex] \ left (x, f (x) \ right) [/ latex], в которых [latex] f (-x) = -f (x) [/ latex] , и есть симметрия относительно начала координат.
      • вертикальная асимптота : прямая линия, параллельная оси [latex] y [/ latex], к которой кривая приближается произвольно близко, когда кривая уходит в бесконечность.
      Построение касательной функции

      Касательная функция может быть построена на графике путем нанесения точек [latex] \ left (x, f (x) \ right) [/ latex]. Форму функции можно создать, найдя значения тангенса под определенными углами. Однако невозможно найти касательные функции для этих особых углов с единичной окружностью.Мы применяем формулу [latex] \ displaystyle {\ tan x = \ frac {\ sin x} {\ cos x}} [/ latex], чтобы определить касательную для каждого значения.

      Мы можем проанализировать графическое поведение касательной функции, посмотрев на значения некоторых специальных углов. Рассмотрим точки ниже, для которых координаты [latex] x [/ latex] представляют собой углы в радианах, а координаты [latex] y [/ latex] — [latex] \ tan x [/ latex]:

      [латекс] \ displaystyle {(- \ frac {\ pi} {2}, \ text {undefined}) \ quad (- \ frac {\ pi} {3}, — \ sqrt {3}) \ quad (- \ frac {\ pi} {4}, -1) \ quad (- \ frac {\ pi} {6}, — \ frac {\ sqrt {3}} {3}) \ quad (0, 0) \\ (\ frac {\ pi} {6}, \ frac {\ sqrt {3}} {3}) \ quad (\ frac {\ pi} {4}, 1) \ quad (\ frac {\ pi} {3 }, \ sqrt {3}) \ quad (\ frac {\ pi} {2}, \ text {undefined})} [/ latex]

      Обратите внимание, что [latex] \ tan x [/ latex] не определено в [latex] \ displaystyle {x = — \ frac {\ pi} {2}} [/ latex] и [latex] \ displaystyle {x = \ frac {\ pi} {2}} [/ латекс].Вышеупомянутые пункты помогут нам нарисовать наш график, но нам нужно определить, как граф ведет себя там, где он не определен. Давайте рассмотрим последние четыре пункта. Мы можем определить, что значения [latex] y [/ latex] увеличиваются по мере того, как [latex] x [/ latex] увеличивается и приближается к [latex] \ displaystyle {\ frac {\ pi} {2}} [/ latex]. Мы могли бы рассмотреть дополнительные точки между [latex] \ displaystyle {x = 0} [/ latex] и [latex] \ displaystyle {x = \ frac {\ pi} {2}} [/ latex], и мы увидим, что это держит. Точно так же мы видим, что [latex] y [/ latex] уменьшается по мере приближения [latex] x [/ latex] к [latex] \ displaystyle {- \ frac {\ pi} {2}} [/ latex], потому что выходные становиться все меньше и меньше.

      Напомним, что существует несколько значений [latex] x [/ latex], которые могут дать [latex] \ cos x = 0 [/ latex]. В любой такой точке [latex] \ tan x [/ latex] не определено, потому что [latex] \ displaystyle {\ tan x = \ frac {\ sin x} {\ cos x}} [/ latex]. При значениях, при которых функция касания не определена, на ее графике наблюдаются разрывы. При этих значениях график касательной имеет вертикальные асимптоты.

      График функции касательной: функция касательной имеет вертикальные асимптоты в [latex] \ displaystyle {x = \ frac {\ pi} {2}} [/ latex] и [latex] \ displaystyle {x = — \ frac {\ пи} {2}} [/ латекс].

      Характеристики графика касательной функции

      Как и функции синуса и косинуса, тангенс является периодической функцией. Это означает, что его значения повторяются через равные промежутки времени. Период касательной функции равен [latex] \ pi [/ latex], потому что график повторяется на [latex] x [/ latex] -осных интервалах [latex] k \ pi [/ latex], где [latex] k [/ latex] — это константа. На графике функции касательной на интервале [latex] \ displaystyle {- \ frac {\ pi} {2}} [/ latex] к [latex] \ displaystyle {\ frac {\ pi} {2}} [/ latex], мы можем увидеть поведение графика за один полный цикл функции.Если мы посмотрим на
      любой больший интервал, мы увидим, что характеристики графика повторяются.

      График функции касательной симметричен относительно начала координат и, следовательно, является нечетной функцией. Другими словами, [latex] \ text {tan} (- x) = — \ text {tan} x [/ latex] для любого значения [latex] x [/ latex]. Любые две точки с противоположными значениями [latex] x [/ latex] производят противоположные значения [latex] y [/ latex]. Мы можем видеть, что это правда, рассматривая значения [latex] y [/ latex] графика при любых противоположных значениях [latex] x [/ latex].Рассмотрим [латекс] \ displaystyle {x = \ frac {\ pi} {3}} [/ latex] и [latex] \ displaystyle {x = — \ frac {\ pi} {3}} [/ latex]. Выше мы уже определили, что [латекс] \ displaystyle {\ tan (\ frac {\ pi} {3}) = \ sqrt {3}} [/ latex] и [latex] \ displaystyle {\ tan (- \ frac { \ pi} {3}) = — \ sqrt {3}} [/ latex].

      Секанс и тригонометрические функции

      Тригонометрические функции имеют обратные величины, которые можно вычислить с помощью единичной окружности.

      Цели обучения

      Расчет значений тригонометрических функций, являющихся обратными синусу, косинусу и тангенсу

      Ключевые выносы

      Ключевые моменты
      • Секущая функция обратна функции косинуса [latex] \ displaystyle {\ left (\ sec x = \ frac {1} {\ cos x} \ right)} [/ latex].Его можно найти для угла [латекс] t [/ latex], используя координату [latex] x [/ latex] связанной точки на единичной окружности: [latex] \ displaystyle {\ sec t = \ frac { 1} {x}} [/ латекс].
      • Функция косеканса является обратной функцией синусоидальной функции [latex] \ displaystyle {\ left (\ csc x = \ frac {1} {\ sin x} \ right)} [/ latex]. Его можно найти для угла [латекс] t [/ latex], используя координату [latex] y [/ latex] связанной точки на единичном круге: [latex] \ displaystyle {\ csc t = \ frac { 1} {y}} [/ латекс].
      • Функция котангенса является обратной функцией касательной [латекс] \ displaystyle {\ left (\ cot x = \ frac {1} {\ tan x} = \ frac {\ cos t} {\ sin t} \ right) }[/латекс]. Его можно найти для угла, используя координаты [latex] x [/ latex] и [latex] y [/ latex] соответствующей точки на единичной окружности: [latex] \ displaystyle {\ cot t = \ frac {\ cos t} {\ sin t} = \ frac {x} {y}} [/ latex].
      Ключевые термины
      • секанс : величина, обратная функции косинуса
      • косеканс : величина, обратная синусоиде
      • .
      • котангенс : величина, обратная касательной функции

      Введение в взаимные функции

      Мы обсудили три тригонометрические функции: синус, косинус и тангенс.Каждая из этих функций имеет обратную функцию, которая определяется обратной величиной отношения исходной тригонометрической функции. Обратите внимание, что обратные функции отличаются от обратных функций. Обратные функции — это способ работы в обратном направлении или определения угла с учетом тригонометрического отношения; они предполагают работу с теми же соотношениями, что и исходная функция.

      Три взаимные функции описаны ниже.

      Секант

      Секущая функция обратна функции косинуса и обозначается сокращенно как [латекс] \ сек [/ латекс].
      Его можно описать как отношение длины гипотенузы к длине соседней стороны в треугольнике.

      [латекс] \ displaystyle {\ begin {align} \ sec x & = \ frac {1} {\ cos x} \\ \ sec x & = \ frac {\ text {hypotenuse}} {\ text {смежный}} \ end {align}} [/ latex]

      Секанс легко вычислить со значениями в единичной окружности. Напомним, что для любой точки круга значение [latex] x [/ latex] дает [latex] \ cos t [/ latex] для соответствующего угла [latex] t [/ latex].Следовательно, секущая функция для этого угла равна

      [латекс] \ displaystyle {\ sec t = \ frac {1} {x}} [/ латекс]

      Косеканс

      Функция косеканса является обратной функцией синусоиды и обозначается сокращенно как [latex] \ csc [/ latex]. Его можно описать как отношение длины гипотенузы к длине противоположной стороны треугольника.

      [латекс] \ displaystyle {\ begin {align} \ csc x & = \ frac {1} {\ sin x} \\ \ csc x & = \ frac {\ text {hypotenuse}} {\ text {напротив}} \ end {align}} [/ latex]

      Как и секанс, косеканс может быть вычислен со значениями в единичной окружности.Напомним, что для любой точки круга значение [latex] y [/ latex] дает [latex] \ sin t [/ latex]. Следовательно, функция косеканса для того же угла равна

      [латекс] \ displaystyle {\ csc t = \ frac {1} {y}} [/ латекс]

      Котангенс

      Функция котангенса обратна функции тангенса и обозначается сокращенно как [latex] \ cot [/ latex]. Его можно описать как отношение длины соседней стороны к длине гипотенузы в треугольнике.

      [латекс] \ displaystyle {\ begin {align} \ cot x & = \ frac {1} {\ tan x} \\ \ cot x & = \ frac {\ text {смежный}} {\ text {противоположный}} \ end {align}} [/ latex]

      Также обратите внимание, что поскольку [latex] \ displaystyle {\ tan x = \ frac {\ sin x} {\ cos x}} [/ latex], его обратная величина равна

      [латекс] \ displaystyle {\ cot x = \ frac {\ cos x} {\ sin x}} [/ latex]

      Котангенс также может быть вычислен со значениями в единичной окружности.Применяя координаты [latex] x [/ latex] и [latex] y [/ latex], связанные с углом [latex] t [/ latex], получаем

      [латекс] \ displaystyle {\ begin {align} \ cot t & = \ frac {\ cos t} {\ sin t} \\ \ cot t & = \ frac {x} {y} \ end {align}} [/ латекс]

      Вычисление взаимных функций

      Теперь мы распознаем шесть тригонометрических функций, которые можно вычислить, используя значения в единичном круге. Напомним, что мы использовали значения функций синуса и косинуса для вычисления функции тангенса.Мы будем следовать аналогичному процессу для обратных функций, ссылаясь на значения в единичном круге для наших расчетов.

      Например, давайте найдем значение [latex] \ sec {\ left (\ frac {\ pi} {3} \ right)} [/ latex].

      Применяя [latex] \ displaystyle {\ sec x = \ frac {1} {\ cos x}} [/ latex], мы можем переписать это как:

      [латекс] \ displaystyle {\ sec {\ left (\ frac {\ pi} {3} \ right)} = \ frac {1} {\ cos {\ left ({\ frac {\ pi} {3}} \ right)}}} [/ латекс]

      Из единичного круга мы знаем, что [латекс] \ displaystyle {\ cos {\ left ({\ frac {\ pi} {3}} \ right)} = \ frac {1} {2}} [/ latex] .Используя это, можно найти значение [latex] \ displaystyle {\ sec {\ left (\ frac {\ pi} {3} \ right)}} [/ latex]:

      [латекс] \ displaystyle {\ begin {align} \ sec {\ left (\ frac {\ pi} {3} \ right)} & = \ frac {1} {\ frac {1} {2}} \\ & = 2 \ end {align}} [/ латекс]

      Остальные взаимные функции могут быть решены аналогичным образом.

      Пример

      Используйте единичный круг для вычисления [латекс] \ sec t [/ latex], [latex] \ cot t [/ latex] и [latex] \ csc t [/ latex] в точке [latex] \ displaystyle {\ left (- \ frac {\ sqrt {3}} {2}, \ frac {1} {2} \ right)} [/ latex].

      Точка на единичном круге: Точка [латекс] \ displaystyle {\ left (- \ frac {\ sqrt {3}} {2}, \ frac {1} {2} \ right)} [/ latex] , показанный в единичном круге.

      Поскольку нам известны координаты [latex] (x, y) [/ latex] точки на единичной окружности, обозначенной углом [latex] t [/ latex], мы можем использовать эти координаты для нахождения трех функций.

      Напомним, что координата [latex] x [/ latex] дает значение для функции косинуса, а координата [latex] y [/ latex] дает значение для функции синуса.Другими словами:

      [латекс] \ displaystyle {\ begin {align} x & = \ cos t \\ & = — \ frac {\ sqrt {3}} {2} \ end {align}} [/ latex]

      и

      [латекс] \ displaystyle {\ begin {align} y & = \ sin t \\ & = \ frac {1} {2} \ end {align}} [/ latex]

      Используя эту информацию, можно вычислить значения обратных функций под углом [латекс] t [/ латекс]:

      [латекс] \ displaystyle {\ begin {align} \ sec t & = \ frac {1} {\ cos t} \\ & = \ frac {1} {x} \\ & = \ left (\ frac {1 } {- \ frac {\ sqrt {3}} {2}} \ right) \\ & = — \ frac {2} {\ sqrt {3}} \\ & = \ left (- \ frac {2} { \ sqrt {3}} \ cdot \ frac {\ sqrt {3}} {\ sqrt {3}} \ right) \\ & = — \ frac {2 \ sqrt {3}} {3} \ end {align} } [/ латекс]

      [латекс] \ displaystyle {\ begin {align} \ cot t & = \ frac {\ cos t} {\ sin t} \\ & = \ frac {x} {y} \\ & = \ left (\ frac {- \ frac {\ sqrt {3}} {2}} {\ frac {1} {2}} \ right) \\ & = \ left (- \ frac {\ sqrt {3}} {2} \ cdot \ frac {2} {1} \ right) \\ & = — \ sqrt {3} \ end {align}} [/ latex]

      [латекс] \ displaystyle {\ begin {align} \ csc t & = \ frac {1} {\ sin t} \\ & = \ frac {1} {y} \\ & = \ left (\ frac {1 } {\ frac {1} {2}} \ right) \\ & = 2 \ end {align}} [/ latex]

      Обратные тригонометрические функции

      Каждая тригонометрическая функция имеет обратную функцию, которую можно изобразить в виде графика. {- 1} x = y [/ latex].{-1} [/ латекс]

    2. однозначная функция : функция, которая никогда не сопоставляет отдельные элементы своего домена с одним и тем же элементом своего диапазона.
    3. Введение в обратные тригонометрические функции

      Обратные тригонометрические функции используются для нахождения углов треугольника, если нам заданы длины сторон. Обратные тригонометрические функции могут использоваться, чтобы определить, какой угол даст определенное значение синуса, косинуса или тангенса.

      Чтобы использовать обратные тригонометрические функции, мы должны понимать, что обратная тригонометрическая функция «отменяет» то, что «делает» исходная тригонометрическая функция, как и в случае с любой другой функцией и ее обратной.{-1} (б) = а [/ латекс]. Однако функции синуса, косинуса и тангенса — это , а не взаимно однозначные функции. График каждой функции не прошел бы тест горизонтальной линии. Фактически, никакая периодическая функция не может быть взаимно однозначной, потому что каждый выход в ее диапазоне соответствует по крайней мере одному входу в каждом периоде, а количество периодов бесконечно. Как и в случае с другими функциями, которые не являются взаимно однозначными, нам нужно будет ограничить область определения каждой функции, чтобы получить новую функцию, которая является взаимно однозначной. Мы выбираем домен для каждой функции, который включает число [latex] 0 [/ latex].

      Функции синуса и косинуса в ограниченных областях: (a) Функция синуса, показанная в ограниченной области [latex] \ left [- \ frac {\ pi} {2}, \ frac {\ pi} {2} \ справа] [/ латекс]; (b) Функция косинуса, показанная в ограниченной области [latex] \ left [0, \ pi \ right] [/ latex].

      График функции синуса ограничен областью [latex] [- \ frac {\ pi} {2}, \ frac {\ pi} {2}] [/ latex] и графиком функции косинуса ограничено [латексом] [0, \ pi] [/ латексом]. График касательной функции ограничен [latex] \ left (- \ frac {\ pi} {2}, \ frac {\ pi} {2} \ right) [/ latex].

      Касательная функция в ограниченной области

      Функция касательной, показанная в ограниченной области [latex] \ left (- \ frac {\ pi} {2}, \ frac {\ pi} {2} \ right) [/ latex].

      Эти варианты выбора ограниченных доменов в некоторой степени произвольны, но они имеют важные полезные характеристики. Каждый домен включает начало координат и некоторые положительные значения, и, что наиболее важно, каждый результат дает взаимно однозначную функцию, которая является обратимой. Традиционный выбор для ограниченной области касательной функции также имеет то полезное свойство, что он простирается от одной вертикальной асимптоты к другой вместо того, чтобы разбиваться на части асимптотой.{-1} x \ quad \ text {имеет домен} \ quad \ left (- \ infty, \ infty \ right) \ quad \ text {и диапазон} \ quad \ left (- \ frac {\ pi} {2} , \ frac {\ pi} {2} \ right)} [/ latex]

      Графики обратных тригонометрических функций

      Функция синуса и функция обратного синуса (или арксинуса): функция арксинуса является отражением функции синуса относительно линии [latex] y = x [/ latex].

      Чтобы найти область определения и диапазон обратных тригонометрических функций, мы меняем область определения и диапазон исходных функций.

      Функция косинуса и функция обратного косинуса (или арккосинуса): Функция арккосинуса является отражением функции косинуса относительно линии [латекс] y = x [/ latex].

      Каждый график обратной тригонометрической функции является отражением графика исходной функции относительно линии [латекс] y = x [/ latex].

      Функция тангенса и функция арктангенса (или арктангенса): Функция арктангенса является отражением функции касательной относительно линии [latex] y = x [/ latex].{-1} х = у [/ латекс].

      ТРИГОНОМЕТРИЯ

      ТРИГОНОМЕТРИЯ
      Щелкните здесь, чтобы найти основные формулы.

      Тригонометрический круг и углы

      Выберите ось x и ось y (ортонормированная) и пусть O будет началом координат.
      Окружность радиуса один с центром в точке O называется «тригонометрический круг» или «единичный круг».
      Поворот против часовой стрелки — положительная ориентация в тригонометрии.
      Углы отсчитываются от оси x.
      Единицы измерения угла — градус и радиан.
      Прямой угол — это угол, размер которого составляет точно 90 градусов или пи / 2 радиана.
      В этой теории мы используем в основном радианы.
      Каждое действительное число t соответствует ровно одному углу и ровно одной точке P на единичной окружности.
      Мы называем эту точку «точкой изображения» t.

      Примеры:

      • пи / 6 соответствует углу t и точке P на окружности.
      • -pi / 2 соответствует углу u и точке Q на окружности.

      Тригонометрические числа действительного числа t

      Действительное число t соответствует ровно одной точке P на единичной окружности.
      • Координата X точки P называется косинусом t. Мы пишем cos (t).
      • Координата Y точки P называется синусом t. Мы пишем sin (t).
      • Число sin (t) / cos (t) называется тангенсом t. Мы пишем tan (t).
      • Число cos (t) / sin (t) называется котангенсом t. Пишем cot (t).
      • Число 1 / cos (t) называется секансом t. Пишем sec (t)
      • Число 1 / sin (t) называется косекансом t. Мы пишем csc (t) или cosec (t)
      Линия с уравнением sin (t).x — cos (t). y = 0
      содержит начало координат и точку P (cos (t), sin (t)). Итак, эта строка — OP.
      На этой прямой мы берем точку пересечения S (1,?) С прямой x = 1.
      Это легко увидеть? = загар (т).
      Итак, tan (t) — координата y точки S.

      Аналогичным образом находим, что cotan (t) — координата x точки пересечения S ‘ линии OP с линией y = 1.

      Основные формулы

      С t радиан соответствует ровно одна точка P (cos (t), sin (t)) на единичной окружности.Квадрат расстояния [OP] = 1. Расчет | OP | 2 , используя координаты P, находим для каждого t:
      cos  2  (t) + sin  2  (t) = 1
      
                          грех  2  (т)
      1 + загар  2  (t) = 1 + ----------
                          cos  2  (т)
      
                   cos  2  (t) + sin  2  (t)
                 знак равно
                       cos  2  (т)
      
      
                        1
                 = ----------- = сек  2  (t)
                     cos  2  (т)
      
      Таким же образом:
      
      1 + котан  2  (t) = 1 / sin  2  (t) = csc  2  (t)
       
             cos  2  (t) + sin  2  (t) = 1
      
             1 + tan  2  (t) = sec  2  (t)
      
             1 + детская кроватка  2  (t) = csc  2  (t)
       

      Примеры использования:
           sin  2  (t) = 1 - cos  2  (t)
      
           cos  2  (4t) = 1 - sin  2  (4t)
      
           1 + tan  2  (t / 2) = sec  2  (t / 2)
      
           csc  2  (t  2 ) - детская кроватка  2  (t  2 ) = 1
       
      Упражнение:
          Если cos (t) = 0.5, то sin  2  (t) = ...
      
          Если cos (t) = 0,1, то tan  2  (t) = ...
      
          Если cot (t) = 0,2, то sin  2  (t) = ...
       

      Связанные значения

      Разница между двумя значениями кратна 2.pi
      Если разница между t и t ‘является целым числом, кратным 2.pi, соответствующие точки на единичной окружности совпадают. Так
      Если t - t '= 2.k.pi (k - целое число)
      потом
             sin (t) = sin (t ') 
      cos (t) = cos (t')
      tan (t) = tan (t ')
      cot (t) = cot (t')

      дополнительные значения

      t и t ‘- дополнительные значения t + t’ = pi.

      С помощью единичного круга видим, что соответствующие точки изображения симметричны относительно ось ординат. Следовательно, мы имеем:

             sin (t) = sin (pi - t) 
      cos (t) = -cos (pi - t)
      tan (t) = -tan (pi - t)
      cot (t) = -cot (pi - t)

      Примеры использования:
         грех (т + пи / 2) = грех (пи / 2 - т)
      
         tan (2t + 0,2) = - tan (pi -0,2 - 2t)
      
         - загар (пи-т) = загар (т)
      
      
           грех (пи-т) + соз (3пи-т) - грех (т + 4пи) + соз (т)
         = sin (t) + cos (pi-t) - sin (t) + cos (t)
         = sin (t) - cos (t) - sin (t) + cos (t)
         = 0
       
      дополнительных значений

      t и t ‘являются дополнительными значениями t + t’ = pi / 2.

      Соответствующие точки изображения на единичной окружности симметричны относительно прямой y = x. Следовательно, мы имеем:

             sin (t) = cos (pi / 2 - t) 
      cos (t) = sin (pi / 2 - t)
      tan (t) = cot (pi / 2 - t)
      cot (t) = tan (pi / 2 - т)

      Примеры использования:
            загар (пи / 4 + 3t) = детская кроватка (пи / 4 -3t)
      
            cos (3pi / 2 -t) = sin (t - pi) = sin (-t + 2pi) = sin (-t)
      
            кроватка (3x - pi / 2) = загар (-3x + pi) = - загар (3x)
      
            - cos (pi / 2 - 2x) + sin (-2x - pi) - cos (3pi - 2x)
          = - sin (2x) + sin (pi - 2x) - cos (pi - 2x)
          = - грех (2х) + грех (2х) + соз (2х)
          = cos (2x)
       
      Противоположные значения

      t и t ‘являются противоположными значениями t + t’ = 0.

      Теперь соответствующие точки изображения симметричны относительно оси x. Следовательно, мы имеем:

             sin (t) = -sin (-t) 
      cos (t) = cos (-t)
      tan (t) = -tan (-t)
      cot (t) = -cot (-t)

      Примеры использования:
            cos (-pi / 2 + x) = cos (pi / 2 - x) = sin (x)
      
            sin (6x - pi) = - sin (pi - 6x) = - sin (6x)
      
            детская кроватка (-x + 4pi) = детская кроватка (-x) = - детская кроватка (x)
       
      Антидополнительные ценности

      t и t ‘являются антидополнительными значениями (t-t’ = pi или t’-t = pi)

      Соответствующие точки изображения симметричны относительно начала координат O.Следовательно, мы имеем:

             sin (t) = -sin (t + pi) 
      cos (t) = -cos (t + pi)
      tan (t) = tan (t + pi)
      cot (t) = cot (t + pi)

      Примеры использования:
            загар (5a + 3pi) = загар (5a + pi) = загар (5a)
      
            детская кроватка (t / 2 + pi / 2) = детская кроватка (t / 2 - pi / 2) = - детская кроватка (pi / 2 - t / 2) = - загар (t / 2)
      
            грех (х + 3 пи) + грех (х) = -син (х) + грех (х) = 0
       

      Прямоугольный треугольник

      Обозначим прямой угол треугольника ABC A.Расстояния | AB |, | BC | и | CA | обычно обозначаются буквами c, a и b. Выберите подходящим образом точку B как центр тригонометрического круга. (см. рисунок).
                         
      Теперь sin (B), cos (B) и 1 прямо пропорциональны b, c и a.
              sin (B) cos (B) 1
              ------ = ------ = ---
                б в а
      
      => sin (B) = b / a cos (B) = c / a tan (B) = b / c
      
      и поскольку углы B и C являются дополнительными углами
      
              cos (C) = b / a sin (C) = c / a tan (C) = c / b
       
      В каждом прямоугольном треугольнике ABC с прямым углом A имеем
              sin (B) = b / a cos (B) = c / a tan (B) = b / c
      
              cos (C) = b / a sin (C) = c / a tan (C) = c / b
       

      Прочие объекты в прямоугольном треугольнике
                       
      • Квадрат гипотенузы равен сумме квадратов двух других сторон.a 2 = b 2 + c 2
      • Высота до гипотенузы прямоугольного треугольника является средней пропорциональной между сегментами, на которые он делит гипотенузу. h 2 = x.y
      • Каждый катет прямоугольного треугольника представляет собой среднее значение, пропорциональное гипотенузе. и его ортогональная проекция на гипотенузу. c 2 = a.x и b 2 = a.y

      Заявки:

      Касательные в точках A и B окружности с центром O и радиусом r, пересекаются в точке P.Хорда AB и прямая OP пересекаются в точке S. Пусть a = | OP | и k = | AB |.
      Выразите k как функцию от r и a.
                       
      В прямоугольном треугольнике АОП: катет прямоугольного треугольника является средним пропорциональна гипотенузе и ее ортогональной проекции на гипотенузу.
           | OA |  2  = | OP | | ОС |.
      
      => | ОС | = г  2  / а
       
      В прямоугольном треугольнике OAS: квадрат гипотенузы равен к сумме квадратов двух других сторон.
           | OA |  2  = | ОС |  2  + | AS |  2 
      
      => r  2  = | OS |  2  + к  2 /4
      
      => r  2  = r  4  / a  2  + k  2 /4
      
      => ....
      
               2р ___________
      => k = ---- \ / a  2  - r  2 
               а
       
      Разделите данный отрезок BC построением точки D. Две части BD и DC должны иметь подходящую длину x и y такую, чтобы произведение x.y равно заданному значению m 2 .

      x + y = | BC | en m 2 = x.y
      м — среднее значение, пропорциональное между x и y
      Мы знаем, что высота до гипотенузы BC прямоугольного треугольника ABC равна средний пропорциональный между сегментами, на которые он делит гипотенузу.

      Ищем прямоугольный треугольник ABC с основанием гипотенуза BC. и с m в качестве высоты.
      Вершина A прямоугольного треугольника расположена на окружности с диаметром BC.

                       
      Этапы строительства:
      1. Нарисуйте полукруг диаметром BC.
      2. Проведите параллель BC на расстоянии m от BC.
      3. Эта параллель дает нам точку А.
      4. Нарисуйте высоту AD от A до BC.

      Площадь треугольника

                   
      Площадь треугольника a.h / 2.
      Но в треугольнике BAH sin (B) = h / c.
      Следовательно, площадь треугольника равна a.c.sin (B) / 2.
      Аналогично, у нас есть, что площадь треугольника
      = b.c.sin (A) / 2 = a.b.sin (C) / 2

      Площадь треугольника ABC = (1/2) a.c.sin (B) = (1/2) b.c.sin (A) = (1/2) a.b.sin (C)

      Вы также можете использовать формулу Герона для вычисления площади треугольника.

      Пусть s = половина окружности треугольника = (a + b + c) / 2.
       

      Площадь треугольника ABC = ______________________________ V s (s - a) (s - b) (s - c)

      Упражнение:
      У треугольника есть стороны длиной 5, 4 и 7.
      Точно начертите треугольник.
      Рассчитайте площадь треугольника по формуле Герона и проверьте результат. измеряя высоту треугольника и вычисляя площадь с этой высотой. Теперь измерьте угол треугольника и вычислите площадь в третий раз. И наоборот, вы можете вычислить углы треугольника, если знаете площадь треугольника.

      Правило синуса

      Площадь треугольника ABC = a.c.sin (B) / 2 = b.c.sin (A) / 2 = a.b.sin (C) / 2
      
      => а.c.sin (B) = b.c.sin (A) = a.b.sin (C)
       
      разделив на a.b.c, получим
                а б в
              ------ = ------ = ------
              грех (A) грех (B) грех (C)
       
      Эта формула называется правилом синусов в треугольнике ABC.

      Пусть R будет радиусом круга с центром O, проходящим через точки A, B и C. Пусть B ‘- вторая точка пересечения BO с окружностью. Угол B ‘ в треугольнике BB’C равно или дополняет A.
      В прямоугольном треугольнике BB’C мы видим, что a = 2R sin (B ‘) = 2R sin (A).
      Таким образом, все дроби правила синуса равны 2R.

      В любом треугольнике ABC имеем
                а б в
              ------ = ------ = ------ = 2R
              грех (A) грех (B) грех (C)
       

      Упражнение:
      У треугольника есть стороны длиной a = 5, b = 4 и c = 7.
      Нарисуйте треугольник точно. Вычислите площадь треугольника по формуле Герона.
      Теперь вычислите угол A по формуле площади (1/2).b.c.sin (А).
      Теперь используйте угол A, чтобы найти радиус R описанной окружности.
      Проверьте результат на своем эскизе.

      Однородное выражение в a, b и c
      Примечание:
      Отношение называется однородным в a, b и c тогда и только тогда, когда это отношение остается в силе. когда мы заменяем a, b и c на кратные r.a, r.b и r.c (r не 0).

      Если выражение между сторонами треугольника однородна по a, b и c, мы получаем эквивалентное выражение, заменяя a, b и c грехом (A), грехом (B) и грехом (C).

      Пример:
      В треугольнике.

              b.sin (A-C) = 3.c.cos (A + C)
      
              sin (B) .sin (A-C) = 3. sin (C) .cos (A + C)
       

      Правило косинусов

      В любом треугольнике ABC имеем
              a  2  = b  2  + c  2  - 2 b c cos (A)
      
              b  2  = c  2  + a  2  - 2 c a cos (B)
      
              c  2  = a  2  + b  2  - 2 a b cos (C)
       

      Доказательство:
      Докажем, что a 2 = b 2 + c 2 — 2 b c cos (A)

      Если угол A прямой, то доказательство очевидно.

      Теперь предположим, что угол A — острый угол.

                
      
          a  2  = h  2  + p  2  (*)
      
          b  2  = h  2  + q  2 
      
              = h  2  + (c - p)  2 
       так,
          h  2  = b  2  - (c - p)  2  (**)
      
      
      Из (*) и (**)
      
         a  2  = b  2  - (c - p)  2  + p  2 
      
             = b  2  - (c  2  - 2 p c + p  2 ) + p  2 
      
             = b  2  - c  2  + 2 p c
      
             = b  2  + c  2  + 2 p c - 2 c  2 
      
             = b  2  + c  2  + 2 c (p - c)
      
             = b  2  + c  2  - 2 c (c - p)
      
             = b  2  + c  2  - 2 c q
      
             = b  2  + c  2  - 2 c b cos (A)
       
      Теперь предположим, что угол A — тупой угол.

      Доказательство проводится так же, как указано выше.
      Нарисуйте новую картинку и поработайте над ней как с упражнением.

      Это правило косинуса также можно доказать с помощью скалярного произведения векторов.
      См. Правило косинуса доказательства

      пи / 3

      Пусть V — точка изображения, соответствующая углу pi / 3 на единичной окружности, и пусть E — точка пересечения этой окружности с положительной осью X.
      Треугольник OVE равносторонний. Следовательно, cos (pi / 3) = 1/2.
      sin  2  (pi / 3) = sqrt (1 - cos  2  (pi / 3)) = sqrt (3) / 2
      
      Итак, sin (pi / 3) = sqrt (3) / 2 и cos (pi / 3) = 1/2.загар (пи / 3) = sqrt (3)
       

      пи / 4

      Пусть V будет точкой изображения, соответствующей углу pi / 4 на единичной окружности. Отсюда очевидно, что cos (pi / 4) = sin (pi / 4) и tan (pi / 4) = 1.
      cos  2  (pi / 4) + sin  2  (pi / 4) = 1 => 2cos  2  (pi / 4) = 1 => cos (pi / 4) = sqrt (1/2)
      
      Итак, cos (pi / 4) = sin (pi / 4) = sqrt (1/2)
      
           загар (пи / 4) = 1
       

      пи / 6

      Из свойств дополнительных углов мы имеем: cos (pi / 6) = sqrt (3) / 2 и sin (pi / 6) = 1/2.
      tan (pi / 6) = 1 / sqrt (3).

      Чехол SSS

      Дано: Три стороны.

      Подставьте все стороны в Правило де косинуса, чтобы вычислить углы.

      Пример: a = 4 b = 5 c = 7

      Правило косинуса дает

      58 = 70 cos (А)
      40 = 56 cos (В)
      -8 = 40 cos (Кл)
      
      А = 34,05 В = 44,41 С = 101,53
       
      Тест: A + B + C = …

      Корпус ASA или AAS

      Дано: два угла и сторона.

      Вычислите третий угол, а затем стороны с помощью правила синуса.

      Пример: a = 4 A = 34 B = 45

      Третий угол равен C = 101
      по правилу синуса.

            4 греха (45)
      б = -------------- = 5.06
             грех (34)
      
      
            4 греха (101)
      с = ------------- = 7,02
             грех (34)
       
      Тест: нарисуйте набросок треугольника

      Корпус SAS

      Дано: две стороны и включенный угол.

      Используйте правило косинуса.

      Пример: b = 5 c = 7 A = 34,05

      Из правила косинуса

      а  2  = 25 + 49-70 cos (34,05) => а = 4
      
      Две другие формулы правила косинуса дают
      
      40 = 56 cos (В)
      -8 = 40 cos (Кл)
      
      В = 44,41 С = 101,53
       
      Тест: A + B + C = …

      Корпус SSA

      Дано: две стороны и не включенный угол.

      Нарисуйте эскиз. Есть три случая.
      1) нет решений
      2) одно решение
      3) два решения

      1. A = 60 b = 5 a = 1

        По эскизу видим, что решений нет.

      2. A = 60 b = 5 a = 7

        Из наброска мы видим, что есть одно решение. Мы используем правило синуса.

            7 5 в
        --------- = -------- = ---------
         грех (60) грех (B) грех (C)
         
        Итак, sin (B) = 0,6186, и это дает нам два дополнительных решения для B.
        Но из нашего эскиза мы знаем, какое значение выбрать. В = 38,21.
        Тогда C = 180 — 38,21 — 60 = 81,79
        и c = 8
      3. A = 60 b = 5 a = 4,5

        Из эскиза мы видим, что есть два решения для B. Мы используем правило синуса.

           4,5 5 в
        --------- = -------- = ---------
         грех (60) грех (B) грех (C)
         
        Итак, sin (B) = 0,96225, и это дает нам два дополнительных решения для B.
        B = 74,2 из 105,8
        Сначала выберите B = 74,2 и сначала вычислите C, а затем c с помощью правила синуса.
        Затем выберите B = 105,8 и сначала вычислите C, а затем c с помощью правила синуса.
        Проверьте результаты по вашему эскизу.

      Синусоидальная функция

      Функция определяется:
      грех: R -> R: x -> грех (х)
       
      называется синусоидальной функцией.
      Изображения ограничены [-1,1], период — 2.pi.
      Мы видим, что диапазон функции равен [-1,1].

      Функция косинуса

      Функция определяется:
      cos: R -> R: x -> cos (x)
       
      называется косинус-функцией.
      Изображения ограничены [-1,1], период — 2.pi.
      Диапазон функции [-1,1].

      Функция тангенса

      Функция определяется:
      загар: R -> R: x -> загар (x)
       
      называется касательной функцией.
      Теперь период равен пи, а изображения не определены в x = (pi / 2) + k.pi
      Диапазон или изображение — R .

      Функция котангенса

      Функция определяется:
      детская кроватка: R -> R: x -> детская кроватка (x)
       
      называется функцией котангенса.
      Период равен пи, а изображения не определены в x = k.pi
      Диапазон или изображение — R .

      Связанные функции и период

      Мы можем подвергнуть предыдущие функции всевозможным преобразованиям. Получаем родственные функции. (см. Влияние преобразования на график функции)

      Пример 1

      y = sin (4x)
      График этой функции возникает из графика sin (x), когда мы сжимаем график sin (x) в направлении оси y с коэффициентом 4.
      Отсюда следует, что период sin (4x) равен pi / 2. Функция y = sin (ax) имеет период 2. pi / a, если a> 0.

      Аналогичные правила применяются к другим тригонометрическим функциям. Таким образом, период tan (x / 3) равен 3.pi.

      Пример 2

      y = sin (x + 5)
      График этой функции получается путем перемещения графика sin (x) на пять единиц влево. Срок не меняется.

      Пример 3

      y = tan (x) +5
      График этой функции получается перемещением графика tan (x) на пять единиц вверх.Срок не меняется.

      Пример 4

      Начнем с y = tan (x). Мы сжимаем график в направлении оси Y с коэффициентом 3. Новая функция у = загар (3x). Сдвигаем график на две единицы вправо. Новая функция y = tan (3 (x-2)). Наконец, мы перемещаем последний график на две единицы вниз. Получаем y = tan (3x -6) -2. Период равен пи / 3.

      Обобщение:

      Период A sin (a x + b) равен 2 pi / | a |
      Период A cos (a x + b) равен 2 pi / | a |
      Период A tan (a x + b) равен pi / | a |
      Период A cot (a x + b) равен pi / | a |
      Период A / sin (a x + b) равен 2 pi / | a |
      Период A / cos (a x + b) равен 2 pi / | a |
      Период A / tan (a x + b) равен pi / | a |
      Период A / cot (a x + b) равен pi / | a |

      Период суммы двух функций

      Если
           f (x) - функция с периодом = a
           g (x) - функция с периодом = b
      Затем
            f (x) + g (x) - функция с периодом = c
         
            Существуют строго положительные и взаимно простые целые числа m и n
            такие, что c = m.a = n.b
       
      Примеры

      sin (2x) имеет период pi, а период cos (3x) — 2pi / 3.
      Теперь c = 2. (pi) = 3. (2pi / 3). Итак, 2 пи — это период sin (2x) + cos (3x)

      sin (pi x) имеет 2 как период, а tan (2 pi x / 7) имеет 7/2 как период.
      Теперь c = 7. (2) = 4. (7/2). Итак, 14 — это период sin (pi x) + tan (2 pi x / 7)

      sin (sqrt (2) x) имеет период pi.sqrt (2), а период cos (2x) — период pi.
      Не существует строго положительных целых чисел m и n таких, что
      m. (Пи.sqrt (2)) = п. (пи). Итак, sin (sqrt (2) x) + cos (2x) НЕ имеет периода!

      sin (x) имеет период 2pi, а период cos (pi x) — 2.
      Не существует строго положительных целых чисел m и n таких, что
      m. (2pi) = n. (2). Итак, sin (x) + cos (pi x) НЕ имеет периода!

      Функция arcsin

      Мы ограничиваем область определения синус-функции до [-pi / 2, pi / 2].
      Теперь это ограничение обратимо, потому что каждое значение изображения в [-1,1] соответствует ровно одному исходное значение в [-pi / 2, pi / 2].
      Функция, обратная этой ограниченной синусоидальной функции, называется арксинусной функцией.
      Мы пишем arcsin (x) или asin (x).
      График y = arcsin (x) является зеркальным отображением ограниченного синусоидального графа относительно прямой y = x.
      Домен [-1,1], диапазон [-pi / 2, pi / 2].

      Функция arccos

      Мы ограничиваем область определения функции косинуса до [0, pi].
      Теперь это ограничение обратимо, потому что каждое значение изображения в [-1,1] соответствует ровно одному исходное значение в [0, пи].
      Функция, обратная этой ограниченной функции косинуса, называется функцией арккосинуса.
      Мы пишем arccos (x) или acos (x).
      График y = arccos (x) является зеркальным отображением графа с ограниченным косинусом относительно прямой y = x.
      Домен [-1,1], диапазон [0, pi].

      Функция arctan

      Мы ограничиваем область определения касательной функции до [-pi / 2, pi / 2].
      Функция, обратная этой ограниченной касательной функции, называется функцией арктангенса.Мы пишем arctan (x) или atan (x). График y = arctan (x) является зеркальным отображением ограниченного касательного графа относительно прямой y = x.
      Домен R , диапазон [-pi / 2, pi / 2].

      Функция arccot ​​

      Мы ограничиваем область определения функции котангенса до [0, pi].
      Функция, обратная этой ограниченной функции котангенса, называется функцией арккотангенса.
      Мы пишем arccot ​​(x) или acot (x).
      График y = arccot ​​(x) является зеркальным отображением ограниченного графа котангенса относительно прямой y = x.
      Домен R , диапазон [0, pi].

      Без периода

      Обратные тригонометрические функции не имеют периода!

      Превращения

      Как и в случае с тригонометрическими функциями, мы можем создавать связанные функции с помощью простых преобразований.

      Пример:
      y = 2.arcsin (x-1) получается путем перемещения графика arcsin (x) на одну единицу вправо, а затем путем перемещения умножение всех изображений на два. Домен [0,2], диапазон [-pi, pi].

      cos (u — v)

      Мы доказываем эту формулу, используя концепцию скалярного произведения двух векторов.(См. Теорию о векторов)
      С u соответствует одна точка P (cos (u), sin (u)) на единичной окружности
      С v соответствует одна точка Q (cos (v), sin (v)) на единичной окружности
      Угол, соответствующая дуге QP на окружности, имеет значение u — v.
      Скалярное произведение P . Q = 1.1.cos (u-v).
      Но, используя координаты, мы также имеем P . Q = cos (u) .cos (v) + sin (u) .sin (v).
      Следовательно,
      cos (u-v) = cos (u) .cos (v) + sin (u) .sin (v)

      Пример:
      cos (pi / 3-2x) = cos (pi / 3) cos (2x) + sin (pi / 3) sin (2x) = 0.5 cos (2x) + 0,5 sqrt (3) sin (2x)
       

      cos (u + v)

      cos (u + v) = cos (u — (-v)) = cos (u) .cos (-v) + sin (u) .sin (-v)
      cos (u + v) = cos (u) .cos (v) -sin (u) .sin (v)

      Пример:
      cos (x + x / 2) + cos (x - x / 2) = cos (x) cos (x / 2) + sin (x) sin (x / 2) + cos (x) cos (x / 2) - грех (х) грех (х / 2)
      
                                  = 2 cos (x) cos (x / 2)
       

      грех (u — v)

      sin (u — v) = cos (pi / 2- (uv)) = cos ((pi / 2-u) + v)
      = cos (pi / 2 — u) .cos (v) -sin (pi / 2 — у).грех (v)
      sin (u — v) = sin (u) .cos (v) -cos (u) .sin (v)

      Пример:
      sin (x - pi / 4) = sin (x) cos (pi / 4) - cos (x) sin (pi / 4) = (sin (x) -cos (x)) / sqrt (2)
       

      грех (u + v)

      sin (u + v) = cos (pi / 2- (u + v)) = cos ((pi / 2-u) -v)
      = cos (pi / 2 — u) .cos (v) + sin ( pi / 2 — u) .sin (v)
      sin (u + v) = sin (u) .cos (v) + cos (u) .sin (v)

      желто-коричневый (u + v)

                  грех (и + v) грех (и).cos (v) + cos (u) .sin (v)
      загар (u + v) = ------------ = ---------------------------
                  cos (u + v) cos (u) .cos (v) -sin (u) .sin (v)
       
      Разделив доминатор и знаменатель на cos (u) .cos (v), получим
                 загар (u) + загар (v)
      загар (u + v) = -----------------
                 1 - загар (и) .тан (в)
       

      Пример:
                    загар (и) + загар (пи / 4) загар (и) + 1 1 + загар (и)
      загар (и + пи / 4) = -------------------- = -------------- = ----- --------
                    1 - загар (и).загар (пи / 4) 1 - загар (и) 1 - загар (и)
       

      загар (u — v)

      Таким же образом у нас есть
                 загар (у) - загар (в)
      загар (u-v) = -----------------
                 1 + загар (и) .тан (в)
       

      грех (2u)

      sin (2u) = sin (u + u) = sin (u) .cos (u) + cos (u) .sin (u) = 2sin (u) .cos (u)
      Примеры
        грех (х) = 2 грех (х / 2) .cos (х / 2)
      
        sin (4x) = 2 sin (2x). cos (2x) = 4 sin (x) cos (x) cos (2x)
      
        12 sin (8x) cos (8x) = 6 sin (16x)
       

      cos (2u)

      cos (2u) = cos (u + u) = cos (u).cos (u) -sin (u) .sin (u) = cos 2 (u) — sin 2 (u)
      cos (2u) = cos 2 (u) — sin 2 (u)

      желто-коричневый (2u)

                 загар (и) + загар (и) 2 загар (и)
      загар (2u) = ------------------ = ---------------
                 1 - загар (и). Тан (и) 1- загар (и) загар (и)
       
                  2 загар (u)
      загар (2u) = -----------
                  1- тан  2  (u)
       

      Пример:
                  1
      детская кроватка (2x) = --------
                 загар (2x)
      
                  1 - желто-коричневый  2  (x)
              знак равно
                    2 загар (х)
       
      1 + cos (2u) = 1 + cos  2  (u) -sin  2  (u) = 2 cos  2  (u)
      
      1 - cos (2u) = 1-cos  2  (u) + sin  2  (u) = 2 sin  2  (u)
      
       
      1 + cos (2u) = 2 cos  2  (u)
      
      1 - cos (2u) = 2 sin  2  (u)
       

      Приложения:

      • Из формул Карно следует, что:
        Период cos (2u) = период cos 2 (u) = период sin 2 (u)
      • Разложите выражение на множители 1 + 2 cos (x) + cos (2x)
              1 + 2 cos (x) + cos (2x)
        
              = 2 cos (x) + (1 + cos (2x))
        
              = 2 cos (x) + 2 cos  2  (x)
        
              = 2 cos (x) (1 + cos (x))
        
              = 2 cos (x) 2 cos  2  (x / 2)
        
              = 4 cos (x) cos  2  (x / 2)
         
        Поскольку 2 пи — период (1 + 2 cos (x) + cos (2x)), отсюда следует, что период cos (x) cos 2 (x / 2) равен 2pi.
      • Найдите период загара 2 (4x)

        Период tan 2 (4x) равен периоду 1 + tan 2 (4x).
        Период 1 + tan 2 (4x) равен периоду 1 / cos 2 (4x).
        Период 1 / cos 2 (4x) равен периоду cos 2 (4x).
        Период cos 2 (4x) равен периоду 0,5 (1 + cos (8x)).
        Период 0,5 (1 + cos (8x)) равен периоду cos (8x).
        И этот период равен пи / 4.

      • В треугольнике ABC стороны a, b, c таковы, что 3a = 7c en 3b = 8c.
        Найдите tan 2 (A / 2) без вычисления A или A / 2.

        Решение:

        О трех сторонах мы знаем:

          а б в
         --- = --- = ---
          7 8 3
        
        Поскольку одинаковые треугольники имеют одинаковые углы, мы можем использовать
        a = 7, b = 8 и c = 3 как стороны треугольника.
        
        Из правила косинуса мы можем написать
        
                    б  2  + с  2  - а  2 
          cos (А) = ------------------ = 1/2
                        2 б в
        
        Теперь воспользуемся формулами Карно
        
          1 - cos (A) 2 sin  2  (A / 2)
          ---------- = -------------- = коричневый  2  (A / 2) = 1/3
          1 + cos (A) 2 cos  2  (A / 2)
        
         
      • Найдите точное значение cos (pi / 12)

        Мы знаем: 1 + cos (2u) = 2 cos 2 (u)
        Теперь возьмем 2u = pi / 6 радиан, тогда u = pi / 12 радиан.

        Теперь точное значение cos (pi / 6) равно sqrt (3) / 2. Мы можем использовать формулу Карно для вычисления cos (pi / 12).

            cos  2  (pi / 12) = (1 + cos (pi / 6)) / 2
        
                        = (1 + sqrt (3) / 2) / 2
        
                        = (2 + sqrt (3)) / 4
        
        Итак, cos (pi / 12) = (1/2). sqrt (2 + sqrt (3))
         
      Из формул Карно имеем
      
      cos (2u) = 2 cos  2  (u) -1
      
      
             2
      = ------------ - 1
         1 + загар  2  (н)
      
      
         1 - желто-коричневый  2  (u)
      знак равно
         1 + загар  2  (н)
      
      
      Мы знаем:
                  2 загар (u)
      загар (2u) = -------------
                1 - желто-коричневый  2  (u)
      
      Следовательно,
      
                   2 загар (u)
      грех (2u) = -----------
                 1 + загар  2  (н)
      
       
      Пусть t = tan (u), тогда
      
      
                 1 - т  2 
      cos (2u) = ---------
                 1 + т  2 
      или
                   1 - желто-коричневый  2  (u)
      cos (2u) = -------------
                   1 + загар  2  (н)
      
                   2т
      грех (2u) = --------
                  1 + т  2 
      или
                   2 загар (u)
      грех (2u) = -----------
                 1 + загар  2  (u)
      
                    2т
      загар (2u) = -------
                  1 - т  2 
      или
                  2 загар (u)
      загар (2u) = -------------
                1 - желто-коричневый  2  (u)
      
       

      Эти три формулы называются t-формулами или формулами полуугла.

      Приложение:
      Уравнение прямой d: y = 3 x + 4.
      u = угол между осью x и этой линией.
      Мы знаем, что tan (u) = 3.

      Линия d ‘является отражением линии y = 3 в d.
      Угол между осью x и линией d ‘равен 2u.
      Наклон линии d ‘желтовато-коричневый (2u).

                    2т 2 загар (и) 2. 3
      загар (2u) = ------- = ------------ = ---------- = - 0,75
                  1 - t  2  1 - желтовато-коричневый  2  (u) 1-9
       
      Мы знаем это
      соз (u + v) = cos (u).cos (v) -sin (u) .sin (v)
      cos (u - v) = cos (u). cos (v) + sin (u) .sin (v)
      sin (u + v) = sin (u) .cos (v) + cos (u) .sin (v)
      sin (u - v) = sin (u) .cos (v) -cos (u) .sin (v)
       
      и отсюда мы имеем
      cos (u + v) + cos (u - v) = 2. cos (u) .cos (v)
      cos (u + v) - cos (u - v) = -2.sin (u) .sin (v)
      sin (u + v) + sin (u - v) = 2. sin (u) .cos (v)
      sin (u + v) - sin (u - v) = 2. cos (u) .sin (v)
       
      Пусть x = u + v и y = u — v
      , тогда u = (1/2) (x + y) и v = (1/2) (x — y)

      Теперь у нас есть

      cos (x) + cos (y) = 2 cos ((1/2) (x + y)) cos ((1/2) (x - y))
      cos (x) - cos (y) = -2 sin ((1/2) (x + y)) sin ((1/2) (x - y))
      sin (x) + sin (y) = 2 sin ((1/2) (x + y)) cos ((1/2) (x - y))
      sin (x) - sin (y) = 2 cos ((1/2) (x + y)) sin ((1/2) (x - y))
       
      Формулы Симпсона
                              х + у х - у
      cos (x) + cos (y) = 2 cos ------ cos -------
                                2 2
      
                               х + у х - у
      cos (x) - cos (y) = -2 грех ------ грех -------
                                2 2
      
                              х + у х - у
      грех (х) + грех (у) = 2 грех ------ соз -------
                                2 2
      
                              х + у х - у
      грех (х) - грех (у) = 2 соз ------ грех -------
                                2 2
       

      Пример: формулы можно использовать для факторизации тригонометрических выражений.
         cos (2x) - cos (2y)
         -----------------
         cos (2x) + cos (2y)
      
         -2 грех (х + у) грех (х-у)
      знак равно
          2 cos (x + y) cos (x-y)
      
      
      = - загар (x + y) загар (x-y)
      
      = загар (у + х) загар (у-х)
       
      Многие из предыдущих формул можно использовать для разложения тригонометрических форм. Этот факторинг может быть осуществлен разными способами. На некоторых примерах мы покажем различные методы факторизации.
      •     sin (2a). (1 + tan  2  (a))
        
          = 2 sin (a) cos (a). (1 / cos  2  (а))
        
          = 2 sin (а) / cos (а)
        
          = 2 загар (а)
         

      •      2 грех (2а) + грех (4а)
        
          = 2 греха (2а) + 2 греха (2а).cos (2a)
        
          = 2 sin (2a). (1 + cos (2a))
        
          = 4 sin (a) cos (a) .2 cos  2  (a)
        
          = 8 sin (а) cos  3  (а)
         

      •     1 - желто-коричневый  4  (а)
        
          = (1 - загар  2  (а)). (1 + загар  2  (а))
        
                  грех  2  (а) 1
          = (1 - -------) --------
                  cos  2  (а) cos  2  (а)
        
            cos (2a)
          знак равно
            cos  4  (а)
         

      •     cos  2  (a) - sin  2  (a) - 2 cos (a) + 1
        
          = cos  2  (a) - 2 cos (a) + cos  2  (a)
        
          = 2 cos  2  (а) - 2 cos (а)
        
          = 2 cos (a) (cos (a) - 1)
        
          = -2 cos (a) (1 - cos (a))
        
          = -2 cos (a) 2 sin  2  (a / 2)
        
          = -4 cos (a) sin  2  (a / 2)
         

      •      грех (2а) (1 + 2 соз (2а)) + 2 грех (3а)
        
          = sin (2a) + 2 sin (2a) cos (2a) + 2 sin (3a)
        
          = грех (2а) + грех (4а) + 2 греха (3а)
        
          = 2 sin (3a) cos (a) + 2 sin (3a)
        
          = 2 sin (3a) (1+ cos (a))
        
          = 4 sin (3a) cos  2  (a / 2)
         

      •     cos  2  (a) -2 cos (a) + cos (2a) + sin  2  (a)
        
          = 1 + cos (2a) - 2 cos (a)
        
          = 2 cos  2  (а) - 2 cos (а)
        
          = 2 cos (a) (cos (a) - 1)
        
          = -2 cos (a) (1 - cos (a))
        
          = -2 cos (a) 2 sin  2  (a / 2)
        
          = -4 cos (a) sin  2  (a / 2)
         

      •     (1 - cos (4a))  2 
           -----------------
            (1 - cos  2  (4a))
        
        
             (1 - cos (4a))  2 
         знак равно
           (1 - cos (4a)) (1 + cos (4a))
        
             (1 - cos (4a))
         знак равно
             (1 + cos (4a))
        
             2 грех  2  (2а)
         знак равно
             2 cos  2  (2a)
        
         = загар  2  (2a)
         

      •     cos  4  (а) - sin  4  (а)
        
          = (cos  2  (a) - sin  2  (a)) (cos  2  (a) + sin  2  (a))
        
          = (cos  2  (a) - sin  2  (a))
        
          = cos (2a)
         

      •     грех (а) + грех (б) + грех (в) - грех (а + б + в)
        
          = 2 sin ((a + b) / 2) cos ((a-b) / 2) + 2 cos ((a + b + 2c) / 2) sin ((-a-b) / 2)
        
          = 2 sin ((a + b) / 2) (cos ((a-b) / 2) - cos ((a + b + 2c) / 2)
        
          = -4 sin ((a + b) / 2) sin ((a + c) / 2) sin ((-b-c) / 2))
        
          = 4 sin ((a + b) / 2) sin ((a + c) / 2) sin ((b + c) / 2))
         

      •     sin  2  (a) - sin  2  (b) - sin  2  (a + b)
        
          = sin  2  (a) - sin  2  (b) - (sin (a) cos (b) + cos (a) sin (b))  2 
        
          = sin  2  (a) - sin  2  (b) - sin  2  (a) cos  2  (b) - cos  2  (a) sin  2  (b)
             - 2 sin (a) sin (b) cos (a) cos (b)
        
          = sin  2  (a) (1- cos  2  (b)) - sin  2  (b) (1 + cos  2  (a)) - 2 sin (a) sin (b) cos ( а) cos (б)
        
          = sin  2  (a) sin  2  (b) - sin  2  (b) (1 + cos  2  (a)) - 2 sin (a) sin (b) cos (a) cos ( б)
        
          = sin  2  (b) (sin  2  (a) -1 - cos  2  (a)) - 2 sin (a) sin (b) cos (a) cos (b)
        
          = -2 sin  2  (b) cos  2  (a) - 2 sin (a) sin (b) cos (a) cos (b)
        
          = -2 sin (b) cos (a) (sin (b) cos (a) sin (a) cos (b))
        
          = -2 cos (a) sin (b) sin (a + b)
         

      •      sin (2b + 2c) (cos (2b) + cos (2c)) - sin (2b) - sin (2c)
        
          = sin (2b + 2c) 2 cos (b + c) cos (b-c) - 2 sin (b + c) cos (b-c)
        
          = 4 sin (b + c) cos (b + c) cos (b + c) cos (b-c) - 2 sin (b + c) cos (b-c)
        
          = 2 sin (b + c) cos (b-c) (2 cos  2  (b + c) - 1)
        
          = 2 sin (b + c) cos (b-c) (2 cos  2  (b + c) - cos  2  (b + c) - sin  2  (b + c))
        
          = 2 sin (b + c) cos (b-c) (cos  2  (b + c) - sin  2  (b + c))
        
          = 2 sin (b + c) cos (b-c) cos (2b + 2c)
         
      Мы знаем это
      соз (u + v) = cos (u).cos (v) -sin (u) .sin (v)
      cos (u - v) = cos (u). cos (v) + sin (u) .sin (v)
      sin (u + v) = sin (u) .cos (v) + cos (u) .sin (v)
      sin (u - v) = sin (u) .cos (v) -cos (u) .sin (v)
      
      Таким образом
      
      cos (u + v) + cos (u - v) = 2. cos (u) .cos (v)
      cos (u + v) - cos (u - v) = -2.sin (u) .sin (v)
      sin (u + v) + sin (u - v) = 2. sin (u) .cos (v)
      sin (u + v) - sin (u - v) = 2. cos (u) .sin (v)
      
      или
      
      2. cos (u) .cos (v) = cos (u + v) + cos (u - v).
      -2.sin (u) .sin (v) = cos (u + v) - cos (u - v)
      2. sin (u) .cos (v) = sin (u + v) + sin (u - v)
      2. cos (u) .sin (v) = sin (u + v) - sin (u - v)
       
      Период cos (u).cos (v) равен периоду cos (u + v) + cos (u — v)
      Период sin (u) .sin (v) равен периоду cos (u + v) — cos (u — v)
      Период sin (u) .cos (v) равен периоду sin (u + v) + sin (u — v)
      Период cos (u) .sin (v) равен периоду sin (u + v) — sin (u — v)

      Примеры:

      Период cos (2x) .sin (x + 3) равен периоду sin (3x + 3) — sin (x-3)
      , и этот период равен 2 пи.

      Период cos (4x) .cos (x / 2) равен периоду cos (9x / 2) + cos (7x / 2)
      , а этот период равен 4pi.

      Общая функция синуса имеет y = a sin (b (x-c)) + d как уравнение,
      с a, b, c неотрицательными, а a и b отличными от нуля.

      Мы можем преобразовать многие уравнения тригонометрических функций к виду общая синусоидальная функция с использованием предыдущих формул.

      Приведем несколько примеров таких преобразований.

      •     у = -3 грех (х)
        
            у = 3 грех (х - пи)
         
      •     у = грех (-2x)
        
            у = - грех (2х)
        
            у = грех (2х - пи)
        
            у = грех 2 (х - пи / 2)
         
      •     у = -4 грех (-3x)
        
            у = 4 грех (3х)
         
      •    у = 2 соз (3х)
        
           у = 2 грех (пи / 2 - 3х)
        
           y = -2 sin (3x -pi / 2)
        
           y = 2 sin (3x - 3pi / 2)
        
           у = 2 грех 3 (х - пи / 2)
         
      •    y = -2 cos (3x-1)
        
           у = -2 грех (пи / 2 -3x + 1)
        
           y = 2 sin (3x - пи / 2 -1)
        
           у = 2 грех 3 (х - (пи / 6 + 1/3))
         
      •    y = cos (3x + 4) - cos (3x-4)
        
           у = -2 грех (3х) грех (4)
        
           y = (-2sin (4)) sin (3x)
         
      •    у = грех (4х-3).cos (4x-3)
        
           у = (1/2) грех (8x-6)
        
           у = (1/2) грех (8 (х- (3/4))
         
      Все функции с уравнением вида y = a sin (u) + b cos (u) можно преобразовать в общую синусоидальную функцию.
      Это преобразование несколько сложнее чем в предыдущих примерах.

      a.sin (u) + b.cos (u) можно представить в виде A.sin (u-u o ).
      Тогда преобразование к общей синусоидальной функции несложно.

         a.sin (u) + b.cos (u)
      
      = a (sin (u) + (b / a) cos (u))
      
            Возьмем u  o  так, чтобы tan (u  o ) = - b / a
      
      = a (sin (u) - tan (u  o ) cos (u))
      
      = (a / cos (u  o )).(sin (u) .cos (u  o ) - sin (u  o ) .cos (u))
      
            Пусть A = (a / cos (u  o ))
      
      = А. грех (u - u  o )
      
       
      Пример
        3 sin (x) - 2 cos (x)
      
      = 3 (sin (x) - (2/3) cos (x))
      
                Пусть tan (u  o ) = 2/3; возьмем u  o  = 0,588
      
      = 3 (sin (x) - загар (u  o ) cos (x))
      
      = (3 / cos (u  o )) (sin (x) cos (u  o ) - cos (x) sin (u  o ))
      
      = 3,6055 sin (x - 0,588)
      
       

      Базовые уравнения

      cos (u) = cos (v)
      С помощью единичного круга легко увидеть, что
       cos (u) = cos (v)
      
       (и = v + k.2pi) или (u = -v + k.2pi)
       
      грех (и) = грех (в)
      С помощью единичного круга легко увидеть, что
       грех (и) = грех (v)
      
       (u = v + 2.k.pi) или (u = pi - v + 2.k.pi)
       
      загар (u) = загар (v)
      С помощью единичного круга легко увидеть, что
       загар (и) = загар (v)
      
       (u = v + k.pi) при условии существования tan (u) и tan (v)
       
      детская кроватка (u) = детская кроватка (v)
      С помощью единичного круга легко увидеть, что
       детская кроватка (u) = детская кроватка (v)
      
       (и = v + k.pi) при условии, что кроватка (u) и кроватка (v) существуют
       

      Приведение к основным уравнениям

      Пример 1
      
      cos (2x) = cos (pi-3x)
      
      2x = (pi-3x) + 2.k.pi или 2x = - (pi-3x) + 2.k'.pi
      
      5x = pi + 2.k.pi или -x = -pi + 2.k'.pi
      
      x = pi / 5 + 2.k.pi / 5 или x = pi - 2.k'.pi
      
       
      Пример 2
      загар (x-pi / 2) = загар (2x)
      
      (x-pi / 2) = 2x + k.pi
      
      -x = пи / 2 + k.pi
      
      x = -pi / 2 - k.pi (для этих значений существуют tan (x-pi / 2) и tan (2x)))
      
       
      Пример 3
      соз (х) = -1/3
      
      cos (x) = cos (1.91)
      
      x = 1.91 + 2.k.pi или x = -1.91 - 2.k.pi
      
       
      Пример 4
      грех (2х) = соз (х-пи / 3)
      
      cos (pi / 2 - 2x) = cos (x-pi / 3)
      
      pi / 2 - 2x = x - pi / 3 + 2.k.pi или pi / 2 - 2x = - x + pi / 3 + 2.k'.pi
      
      -3x = - pi / 2 - pi / 3 + 2.k.pi или -x = -pi / 2 + pi / 3 + 2.k'.pi
      
      x = pi / 6 + pi / 9 + 2.k.pi / 3 или x = pi / 2 - pi / 3 - 2.k'.pi
      
      x = 5pi / 18 + 2.k.pi / 3 или x = pi / 6 - 2.k'.pi
      
       
      Пример 5
      3 грех (2х) = соз (2х)
      
      3 загар (2x) = 1
      
      загар (2x) = 1/3
      
      загар (2x) = загар (0,32)
      
      2x = 0,32 + k пи
      
      х = 0,16 + к пи / 2
       
      Пример 6
        загар (2x).детская кроватка (x + pi / 2) = 1
      
        загар (2x) = загар (х + пи / 2)
      
         2x = x + pi / 2 + k.pi
      
         x = pi / 2 + k.pi (при условии, что tan (2x) и cot (x + pi / 2) существуют)
       
      Но детская кроватка (x + pi / 2) не существует для x = pi / 2 + k.pi !!!!!

      Итак, загар (2x). cot (x + pi / 2) = 1 не имеет решений !

      Выражение «при условии…» не является лишним!

      Использование дополнительного неизвестного

      Пример 1
      2sin  2  (2x) + sin (2x) -1 = 0
      
                   (пусть t = sin (2x))
      
      2т  2  + т - 1 = 0
      
      
      
      т = 0.5 или t = -1
      
      
      
      sin (2x) = 0,5 или sin (2x) = -1
      
      
      
      sin (2x) = sin (pi / 6) или sin (2x) = sin (-pi / 2)
      
      
      
      2x = pi / 6 + 2.k.pi или 2x = pi - pi / 6 + 2.k.pi или
              2x = -pi / 2 + 2.k.pi или 2x = pi + pi / 2 + 2.k.pi
      
      
      x = pi / 12 + k.pi или x = 5pi / 12 + k.pi или
              x = -pi / 4 + k.pi или x = 3pi / 4 + k.pi
      
       
      Иногда удобно рассматривать эти решения на единичном круге.

      Пример 2

           cos 10x + 7 = 8 cos 5x
      
           cos 10x - 8 cos 5x + 7 = 0
      
           1 + cos 10x - 8 cos 5x + 6 = 0
      
           2 cos  2  5x - 8 cos 5x + 6 = 0
      
      
           cos  2  5x - 4 cos 5x + 3 = 0
      
      Пусть t = cos 5x
      
          т  2  - 4 т + 3 = 0
      
           t = 3 или t = 1
      
           cos 5x = 1
      
           cos 5x = cos 0
      
           5x = 2kpi
      
           x = 2kpi / 5
       
      Примеры
      Таким же образом следующие уравнения могут быть решены с использованием дополнительных неизвестных.
      загар  2  (3x) + загар (3x) = 0
      
      грех  2  (х) (грех (х) +1) -0,25 (грех (х) +1) = 0
      
      cos (2x) + sin  2  (x) = 0,5
      
      
      загар (2x)-детская кроватка (2x) = 1
      
       
      Проверьте свои результаты, построив графики.

      Использование факторинга

      Пример 1
      3.sin (2x) -2.sin (x) = 0
      
      
      
      6sin (x) cos (x) -2.sin (x) = 0
      
      
      
      2. sin (x). (3cos () - 1) = 0
      
      
      
      sin (x) = 0 или cos (x) = 1/3
      
      
      
      x = k.pi или x = 1,23 + 2 k.pi или x = -1,23 + 2 k'.pi
      
       
      Примеры
      Таким же образом можно решить следующие уравнения, используя факторинг.
      загар (x) загар (4x) + загар  2  (x) = 0
      
      грех (7x) -sin (x) = грех (3x)
      
      соз (4х) + соз (2х) + соз (х) = 0
      
      грех (5x) + грех (3x) = cos (2x) -cos (6x)
       
      Проверьте свои результаты, построив графики.

      Уравнение a.sin (u) + b.cos (u) = c

      Первый метод
      Сначала мы покажем, что a.sin (u) + b.cos (u) может быть преобразован в форму
      A.sin (u-u o ) или в форму A.cos (u-u o ).
         a.sin (u) + b.cos (u)
      
      = a (sin (u) + (b / a) cos (u))
      
            Возьмем u  o  так, чтобы tan (u  o ) = - b / a
      
      = a (sin (u) - tan (u  o ) cos (u))
      
      = (a / cos (u  o )).(sin (u) .cos (u  o ) - sin (u  o ) .cos (u))
      
            Пусть A = (a / cos (u  o ))
      
      = А. грех (u - u  o )
      
      = А. cos (pi / 2 - u + u  o )
      
      = А. cos (u - u  o  ')
       
      Пример
        3 sin (x) - 2 cos (x)
      
      = 3 (sin (x) - (2/3) cos (x))
      
                Пусть tan (u  o ) = 2/3; возьмем u  o  = 0,588
      
      = 3 (sin (x) - загар (u  o ) cos (x))
      
      = (3 / cos (u  o )) (sin (x) cos (u  o ) - cos (x) sin (u  o ))
      
      = 3.6055 грех (х - 0,588)
      
          или
      
      = 3,6055 cos (x - 2,1598)
       
      Постройте график 3 sin (x) — 2 cos (x) и график 3,6055 sin (x — 0,588)

      С помощью этого метода мы можем решить уравнение
      a.sin (u) + b.cos (u) = c

      Пример

      3. грех (2x) + 4. cos (2x) = 2
      
      
      
      грех (2x) + 4/3. cos (2x) = 2/3
                      Пусть tan (t) = 4/3
      
      
      грех (2x) + загар (t). cos (2x) = 2/3
      
      
      
      sin (2x) cos (t) + cos (2x) sin (t) = 2 / 3. cos (t)
      
      
      
      sin (2x + t) = 2 / 3. cos (t)
      
                      поскольку 2 / 3.cos (t) = 0,4
      
      
      грех (2x + 0.927) = грех (0,39)
      
      
      
      2x + 0,927 = 0,39 + 2k.pi или 2x + 0,927 = pi - 0,39 + 2k'.pi
      
      
      
      ....
      
       
      Второй метод
      Используя t-формулы

      Пример

        3 sin (2x) + 4 cos (2x) = 2
                         Пусть tan (x) = t
      
           2 т 1 - т  2 
        3 ------- + 4 -------- = 2
          1 + т  2  1 + т  2 
      
        6 т + 4-4 т  2  = 2 + 2 т  2 
         6 т  2 -6 т - 2 = 0
      
         3 т  2 -3 т -1 = 0
      
         т = 1.26 или t = -0,26
      
         tan (x) = 1,26 или tan (x) = -0,26
      
          x = 0,9 + k pi или x = -0,26 + k pi
       

      Однородные уравнения

      Уравнение однородно по a и b тогда и только тогда, когда мы получаем эквивалентное уравнение при замене a и b на ra и rb (r не 0). Пример: a 3 x 2 +5 a.b 2 x +3 a 2 .b = 0 — уравнение относительно x, однородное по a en b.

      Теперь мы имеем в виду уравнения, однородные по sin (u) и cos (u).
      Процедура

      1. Приведите уравнение к виду F = 0. Если возможно, используйте факторинг влево и решите простые части.
      2. Разделите оставшееся уравнение на подходящую степень cos (u) так, чтобы tan (u) появлялся везде.
      3. Пусть t = tan (u), и решим алгебраическое уравнение.
      4. Вернуться к загар (u)
      Пример
      2.cos  3  (x) + 2.sin  2  (x) cos (x) = 5.sin (x) cos  2  (x)
      
      
      
      соз (х).(2.cos  2  (x) + 2.sin  2  (x) - 5.sin (x) cos (x)) = 0
      
      
      
      Простая часть cos (x) = 0 дает нам x = pi / 2 + k.pi
      
      
      Во второй части мы разделим обе части на cos  2  (x). Тогда у нас есть
      
       2. tan  2  (x) - 5. tan (x) +2 = 0
      
                      Пусть t = tan (x)
      
      
       2. т  2  - 5 т + 2 = 0
      
      
      
      t = 0,5 или t = 2
      
      
      
      tan (x) = 0,5 или tan (x) = 2
      
      
      
      x = 0,464 + k.pi или x = 1,107 + k.pi
      
       

      Другие уравнения

      Некоторые уравнения могут быть решены соответствующим образом, комбинируя различные обсуждаемые методы.Кроме того, некоторые тригонометрические формулы часто используются для преобразования уравнения в подходящая форма. Более того, опыт и понимание играют важную роль при решении сложные уравнения. Приведем неочевидный пример.
      1 / sin (x) + 1 / cos (x) = sqrt (3)
           1 / sin (x) + 1 / cos (x) = sqrt (3)
      
              грех (х) + соз (х)
         ------------------- = sqrt (3)
                грех (х) соз (х)
      
          sin (x) + cos (x) = sqrt (3) sin (x) cos (x) (1)
      
      -------------------------------------------------- ------
       
      Если возвести обе части уравнения (1) в квадрат, получится только произведение sin (x) cos (x).Затем мы можем найти значение sin (x) cos (x), и с его помощью мы упростим уравнение (1).
           Из (1) следует
      
           (sin (x) + cos (x))  2  = 3 (sin (x) cos (x))  2  1 + 2 sin (x) cos (x) = 3 (sin (x) cos (x ))  2 
      
           Пусть y = sin (x) cos (x)
      
         3 года  2 -2 года -1 = 0
      
         у = 1 или -1/3
      
        sin (x) cos (x) = 1 или sin (x) cos (x) = -1/3
      
      Если sin (x) cos (x) = 1, то sin (2x) = 2, а это невозможно.
      
      Заключение: Из (1) следует, что
      
                        грех (х) соз (х) = -1/3 (2)
      
      -------------------------------------------------- --------
      
      Теперь воспользуемся этим результатом.Приведем (2) в (1)
      
                грех (х) + соз (х) = - sqrt (3) / 3 (3)
      
        Если (1) истинно, то (2) истинно и, следовательно, (3) истинно. 
      
      Теперь мы покажем, что верно обратное. Начнем с (3).
      
                грех (х) + соз (х) = - sqrt (3) / 3
      
      => (sin (x) + cos (x)  2  = 1/3
      
      => 1 + 2 sin (x) cos (x) = 1/3
      
      => sin (x) cos (x) = -1/3 и это (2)
      
        Итак, если (3) истинно, то (2) истинно и, следовательно, (1) истинно. 
      
      Вывод: 
      (1) и (3) - эквивалентные уравнения.Решим (3) сейчас. -------------------------------------------------- ----------- грех (х) + соз (х) = - sqrt (3) / 3 cos (pi / 2 -x) + cos (x) = - sqrt (3) / 3 2 cos (pi / 4) cos (pi / 4-x) = - sqrt (3) / 3 sqrt (2) cos (pi / 4-x) = - sqrt (3) / 3 cos (пи / 4-х) = -1 / sqrt (6) Пусть a = arccos (-1 / sqrt (6)) соз (пи / 4-х) = соз (а) pi / 4 - x = a + 2 k pi или pi / 4 - x = -a + 2 k pi x = pi / 4 - a + 2 k pi или x = pi / 4 + a + 2 k pi Решениями данного уравнения являются значения pi / 4 - a + 2 k pi en pi / 4 + a + 2 k pi с a = arccos (-1 / sqrt (6))
      Второй метод

      Многие уравнения можно решить по-разному.Мы покажем альтернативный способ решения уравнение

           1 / sin (x) + 1 / cos (x) = sqrt (3)
       
      Период функции в левой части равен 2 пи. Если у нас есть решения уравнения в [0, 2pi], то мы знаем все решения.

      Сначала мы вычисляем решения в [0, 2pi]. Поскольку правая часть уравнения положительна, решения возможны только тогда, когда левая часть также положительна.

      Построив график функции 1 / sin (x) + 1 / cos (x), мы видим, что изображение положительное. в интервалах (0, пи / 2); (3pi / 4, pi) и (3pi / 2, 7pi / 4).

      Решения могут возникать только в этих интервалах. Если ограничить значения x этими интервалами, то обе части уравнения положительные, и мы можем написать

           1 / sin (x) + 1 / cos (x) = sqrt (3)
      
        (1 / sin (x) + 1 / cos (x))  2  = 3
      
              грех (х) + соз (х)
         (------------------)  2  = 3
                грех (х) соз (х)
      
         1 + 2 sin (x) cos (x) = 3 sin  2  (x) cos  2  (x)
      
                   Пусть y = sin (x) cos (x)
      
          3 года  2 -2 года - 1 = 0
      
           y = 1 или y = -1/3
      
      Случай y = 1
      
             грех (х) соз (х) = 1
      
           2 грех (х) соз (х) = 2
      
            грех (2x) = 2
      
         В этом случае решений нет.
      
      Случай y = -1/3
      
              2 sin (x) cos (x) = -2/3
      
           грех (2x) = -2/3
      
                   пусть b = arcsin (-2/3); б = -0.7297
      
           грех (2x) = грех (б)
      
           2x = b + 2 k pi или 2x = (pi-b) + 2kpi
      
            x = b / 2 + k pi или x = pi / 2 - b / 2 + k pi
      
      
      Теперь мы возьмем только значения x, расположенные в интервалах
      (0, пи / 2); (3pi / 4, pi) en (3pi / 2, 7pi / 4).
      
      Есть 2 решения:
      
            х = b / 2 + пи = 2,7767
      и
            х = пи / 2 - b / 2 + пи = 3pi / 2 - b / 2 = 5,077
      
      Все решения уравнения 1 / sin (x) + 1 / cos (x) = sqrt (3) являются
      
        b / 2 + pi + 2 k pi en 3pi / 2 - b / 2 + 2 k pi
       

      Конвенции

      k — целое число.

      знак равно ‘> =’ означает больше или равно

      Примеры

      • грех (x / 2)> 1/2

        Нарисуем решения для (x / 2) на единичной окружности.

        Теперь мы видим, что:

            грех (х / 2)> 1/2
        
          пи / 6 + 2 к пи пи / 3 +4 к пи
        Для каждого k у нас есть открытый интервал с решениями. 
        Множество решений V представляет собой объединение всех этих открытых интервалов.
        V = { U  (pi / 3 + 4 k pi, 5 pi / 3 + 4 k pi)  |  k в  Z }
               k
         
      • желто-коричневый (2x) Нарисуем решения для (2x) на единичной окружности.

        Теперь мы видим, что:

            загар (2x) -pi / 2 + k pi -pi / 4 + k pi / 2
        Для каждого k у нас есть открытый интервал с решениями. 
        Множество решений V представляет собой объединение всех этих открытых интервалов.
        V = { U  (-pi / 4 + k pi / 2, 0,16 + k pi / 2)  |  k в  Z }
               k
         
      • коричневый (2x + pi / 5) Это вариант предыдущего примера.

        Цифра такая же, как и предыдущая, но теперь дает решения для (2x + pi / 5).
        Теперь у нас есть:

                 tan (2x + pi / 5) -pi / 2 + k pi -pi / 2 -pi / 5 + k pi -7 pi / 20 + k pi / 2
        Набор решений V:
        
        
         
        V = { U  (-7 пи / 20 + k пи / 2, 0,16 - пи / 10 + k пи / 2)  |  k в  Z }
               k
         
      • 2 sin 2 (x) — 3 sin (x) + 1 = Пусть t = sin (x).
               2 t  2  - 3 t + 1 2 (t - 1) (t - 1/2) 1/2 = 1/2 = pi / 6 + 2 k pi =
        Набор решений
         
        V = { U  [пи / 6 + 2 k пи; 5pi / 6 + 2 k pi]  |  k в  Z }
               k
         
      • Другой метод решения 2 sin 2 (x) — 3 sin (x) + 1 = Можно также напрямую разложить левую часть на множители и исследовать знак в интервале периода.
                 2 грех  2  (х) - 3 грех (х) + 1
        
              2 (грех (х) - 1) (грех (х) - 1/2)
         
        Возьмем простой период-интервал [0,2pi). Исследуем знак каждого фактора.
            x 0 пи / 6 пи / 2 5 пи / 6 пи 2 пи
        -------------------------------------------------- -------------
         sin (x) -1 - - - - - - 0 - - - - - - - - - - - - - - -
        -------------------------------------------------- -------------
        sin (x) -1/2 - - 0 + + + + + + 0 - - - - - - - - - -
        -------------------------------------------------- -------------
         продукт + + 0 - - - 0 - - 0 + + + + + + + + + +
        -------------------------------------------------- -------------
         
        Решения в интервале периодов равны
        пи / 6 = Набор решений:
        V = { U  [пи / 6 + 2 k пи; 5pi / 6 + 2 k pi]  |  k в  Z }
               k
         
      • косек (x) Сначала исследуем неравенство в интервале периода [0,2pi).

        Значения 0; пи / 2; Пи ; 3pi / 2 не являются решениями. Исследуем другие ценности x в каждом квадранте.

        • Первый квадрант
                cosec (x) 1 / sin (x) 0
          
               cos (x)
          Набор решений (пи / 4, пи / 2).
           
        • Второй квадрант

          Теперь у нас есть cos (x) 0. Решений нет.

        • Третий квадрант
                cosec (x) 1 / sin (x) 0
          
               cos (x)
          Набор решений равен (пи, 5 пи / 4)
          
           
        • Четвертый квадрант Теперь у нас есть cos (x)> 0 и sin (x) Набор решений (3 пи / 2, 2 пи)
        Множество решений данного неравенства представляет собой объединение всех интервалов
           (пи / 4 + 2 к пи, пи / 2 + 2 к пи)
           (пи + 2 к пи, 5 пи / 4 + 2 к пи)
           (3pi / 2 + 2 пи, 2 пи + 2 пи)
        
        с k в  Z 
        
         
      • Сначала преобразовать, затем решить.
            детская кроватка (x) + 1
           -------------> 0
               грех (х)
        
        
            детская кроватка (x) + 1
         -------------> 0 и sin (x) не 0
               грех (х)
        
        
               соз (х) + грех (х)
          -------------------> 0 и sin (x) не 0
                  грех  2  (х)
        
        
            cos (x) + sin (x)> 0 и sin (x) не 0
        
        
           sin (x) + sin (pi / 2 -x)> 0 и sin (x) не 0
        
                    с формулами Симпсона
        
           2 sin (pi / 4) cos (x - pi / 4)> 0
        
           cos (x - pi / 4)> 0 и sin (x) не 0
         
        Используя единичный круг, мы видим, что
           -pi / 2 + 2k пи -pi / 4 + 2k пи
        Множество решений V представляет собой объединение открытых интервалов
         
          V = { U  (-pi / 4 + 2k pi, 3pi / 4 + 2k pi)  |  k дюйм  Z } \ {k pi  |  k в  Z }
                 k
         
      Все уравнения решаются одним и тем же методом.Мы последовательно заменяем уравнение необходимым условием . Это означает, что найденные нами значения не являются необходимыми решениями. После этого мы должны сравнить эти значения с исходным уравнением. Ложные или паразитные значения необходимо удалить.

      Пример 1

           arcsin (2x) = pi / 4 + arcsin (x)
      
           / arcsin (2x) = а
           | arcsin (x) = b (1)
           \ а = пи / 4 + Ь
      
      
          / sin (а) = 2x
      => | грех (б) = х
          \ а = пи / 4 + Ь
      
      
      => / sin (pi / 4 + b) = 2x
          \ грех (Ь) = х
                              формулы сумм
      
      => / cos (b) + sin (b) = 2x.sqrt (2)
          \ грех (Ь) = х
      
      
      => / cos (b) = 2x.sqrt (2) - x
          \ грех (Ь) = х
      
      
      => (2x.sqrt (2) - x)  2  + x  2  = 1
      
      => ....
      
      => x = +0,4798 или x = -0,4798
       
      Мы проверяем эти значения по исходному уравнению. Единственное решение — 0,4798.
      Другое значение x ложно или паразитно.

      Пример 2

         arctan (x + 1) = 3. arctan (x-1)
      
      
         / arctan (x + 1) = а
         | arctan (x-1) = b
         \ а = 3 б
      
      
      => / загар (а) = х + 1
         | загар (б) = х - 1
         \ а = 3 б
      
      => / загар (3b) = х + 1
         \ tan (b) = х - 1
      
                           3 желто-коричневый (б) - коричневый  3  (б)
         но загар (3b) = --------------------
                           1 - 3.загар  2  (б)
      
      
                 3 (х-1) - (х-1)  3 
      => х + 1 = --------------------
                 1-3 (х-1)  2 
      
      
      => (x + 1) (1-3 (x-1)  2 ) = 3 (x-1) - (x-1)  3 
      
      
      => ...
      
      => x = 0 или x = sqrt (2) или x = -sqrt (2)
       
      Мы проверяем эти значения по исходному уравнению. Единственное решение — sqrt (2).
      Другие значения x являются ложными или паразитными.

      Пример 3

         арктангенс (х) + арктангенс (2х) = пи / 4
      
      
      
         / arctan (x) = а
         | arctan (2x) = b
         \ а + Ь = пи / 4
      
      
      => / x = загар (а)
         | 2x = загар (б)
         \ а + Ь = пи / 4
      
      => / x = загар (а)
         \ 2x = загар (пи / 4-а)
      
                                     1 - загар (а)
              но tan (pi / 4-a) = ---------------- поскольку tan (pi / 4) = 1
                                     1 + загар (а)
      
                 1 - х
      => 2x = ----------
                 1 + х
      
      
      =>...
      
      
      => x = (-3 + sqrt (17)) / 4 или x = (-3-sqrt (17)) / 4
       
      Мы проверяем эти значения по исходному уравнению. Единственное решение — (-3 + sqrt (17)) / 4.
      Другое значение x ложно или паразитно.

      Пример 4

       arctan ((x + 1) / (x + 2)) - arctan ((x-1) / (x-2)) = arccos (3 / sqrt (13))
      
      
          / arctan ((x + 1) / (x + 2)) = a
       | arctan ((x-1) / (x-2)) = b
          | arccos (3 / sqrt (13)) = c
          \ а - Ь = с
      
      
          / загар (а) = (х + 1) / (х + 2)
       => | загар (б) = (х-1) / (х-2)
          | cos (c) = 3 / sqrt (13)
          \ а - Ь = с
      
                               загар (а) - загар (б)
             но tan (a-b) = ------------------ и после вычисления находим
                               1 + загар (а) загар (б)
      
      
                          -2 х
           / tan (c) = ----------------
      => | 2 х  2  - 5
           |
           \ cos (c) = 3 / sqrt (13)
      
              Из последнего уравнения следует
               1 + tan  2  (c) = 1 / cos  2  (c) = 13/9 => tan  2  (c) = 4/9
               Сейчас два случая
      
      Первый случай:
                           -2 х
           / tan (c) = ----------------
      => | 2 х  2  - 5
           |
           \ tan (c) = 2/3
      
      
      =>....
      
      => x = 1 или x = -5/2
      
      
      Второй случай:
      
                           -2 х
           / tan (c) = ----------------
      => | 2 х  2  - 5
           |
           \ tan (c) = - 2/3
      
      
      => ....
      
      => x = -1 или x = 5/2
      
       
      Мы проверяем эти значения по исходному уравнению. Единственные решения — 1 и -5/2.
      Другие значения x являются ложными или паразитными. В следующих примерах «меньше или равно» записывается как «= Пример 1
                                        ________
                                       | 2
                                      \ | 1 - п
      Покажи эту кроватку (arcsin (p)) = -----------
                                          п
       
              пусть b = arcsin (p), тогда sin (b) = p с b в [-pi / 2, pi / 2].Итак, cos (b) = sqrt (1 - p  2 ) и
                                            ________
                                           | 2
                                          \ | 1 - п
              детская кроватка (arcsin (p)) = детская кроватка (b) = ---------
                                               п
      
       
      Пример 2
      Докажите, что следующее уравнение не имеет решений при x> 0.
         cos (arctan (x)) + x sin (arctan (x)) = x
       
        Пусть u = arctan (x); Поскольку x> 0 - это u в (0, pi / 2) и tan (u) = x
      
           1 + загар  2  (u) = 1 + x  2 
           1 / cos  2  (u) = 1 + x  2 
            cos (u) = 1 / sqrt (1 + x  2 )
      
       - - - - - - - - - - - - - - - -
      
            грех (и) = загар (и).cos (u)
      
         sin (u) = x / sqrt (1 + x  2 )
      
       - - - - - - - - - - - - - - - -
      
           cos (arctan (x)) + x sin (arctan (x)) = x
      
           cos (u) + x sin (u) = x
      
           1 / sqrt (1 + x  2 ) + x  2  / sqrt (1 + x  2 ) = x
      
            1 + х  2  = х. sqrt (1 + x  2 )
      
            (1 + x  2 )  2  = x  2 . (1 + x  2 )
      
            1 + х  2  = х  2 
      
          и это уравнение не имеет решений.
       

      Пример 3

      Найдите область arccos (arcsin (x))
         x принадлежит области arccos (arcsin (x))
      
           - 1 =
            грех (-1) =
      Вывод: домен arccos (arcsin (x)) равен [-sin (1), sin (1)]
       

      Пример 4

      Найдите домен arcsin (arccos (x))
             x принадлежит области arcsin (arccos (x))
      
             - 1 =
              0 =
               cos (0)> = x> = cos (1)
       
      Вывод: домен arcsin (arccos (x)) равен [cos (1), 1]

      Пример 5

      f (x) = arccos (а.arcsin (x)) с a> 0

      Найдите такие значения a, чтобы область D функции f (x) была как можно больше.

      Вычислите значение a так, чтобы домен D = [-0,5; 0,5]

             x принадлежит области arccos (a. arcsin (x))
      
             - 1 =
             -1 / а = пи / 2
      
             Теперь (*) эквивалентно
      
              -pi / 2 =
               грех (1 / а) = 1/2
      
                а = 6 / пи
       
      Примечание: вы можете проиллюстрировать и изучить многие из предыдущих шагов. с функцией плоттера.

      Пример 6

      Докажите, что для всех натуральных чисел n справедливо следующее выражение.
                      1
          arctan -------------- = arctan (1 / n) - arctan (1 / (n + 1))
                  п  2  + п + 1
       

      Прежде всего отметим, что
                      1 1 1
            0 2 + п + 1 п + 1 п
      
      и поэтому
      
                            1 1 1
            0 2 + п + 1 п + 1 п
      
      
      Это означает, что выражения
                        1
            arctan ------------- и arctan (1 / n) - arctan (1 / (n + 1))
                    п  2  + п + 1
      
      находятся в (0, pi / 4] для всех n.Следовательно, выражение
                      1
          arctan -------------- = arctan (1 / n) - arctan (1 / (n + 1))
                  п  2  + п + 1
      
      эквивалентно
      
               1
         ------------- = загар (arctan (1 / n) - arctan (1 / (n + 1)))
           п  2  + п + 1
      
      Упрощаем правую сторону
      
           загар (arctan (1 / n) - arctan (1 / (n + 1)))
      
           tan (arctan (1 / n)) - загар (arctan (1 / (n + 1)))
      знак равно
          1 + загар (arctan (1 / n)). загар (arctan (1 / (n + 1)))
      
      
              1 / п - 1 / (п + 1)
      знак равно
             1 + (1 / n) (1 / (n + 1))
      
                 1
      знак равно
             п  2  + п + 1
       
      Примечание: вы можете проиллюстрировать и изучить многие из предыдущих шагов с помощью плоттера функций.Щелкните здесь, чтобы найти основные формулы.
      Темы и проблемы

      Домашняя страница MATH-изобилие — урок

      Указатель MATH-учебника

      Условия копирования

      Все предложения, замечания и отчеты об ошибках направлять на jcinfo@telenet.